SlideShare a Scribd company logo
1 of 49
Viết thuê đề tài giá rẻ trọn gói - KB Zalo/Tele : 0973.287.149
Luanvanmaster.com – Cần Kham Thảo - Kết bạn Zalo/Tele : 0973.287.149
ĐẠI HỌC THÁI NGUYÊN
TRƯỜNG ĐẠI HỌC KHOA HỌC
BÙI HỮU MÊN
MỘT SỐ LỚP PHƯƠNG TRÌNH DIOPHANTINE
LUẬN VĂN THẠC SĨ TOÁN HỌC
Thái Nguyên
Viết thuê đề tài giá rẻ trọn gói - KB Zalo/Tele : 0973.287.149
Luanvanmaster.com – Cần Kham Thảo - Kết bạn Zalo/Tele : 0973.287.149
ĐẠI HỌC THÁI NGUYÊN
TRƯỜNG ĐẠI HỌC KHOA HỌC
BÙI HỮU MÊN
MỘT SỐ LỚP PHƯƠNG TRÌNH DIOPHANTINE
LUẬN VĂN THẠC SĨ TOÁN HỌC
Chuyên ngành:Phương pháp Toán sơ cấp
Mã số: 60 46 01 13
NGƯỜI HƯỚNG DẪN KHOA HỌC
GS.TSKH. ĐẶNG HÙNG THẮNG
Thái Nguyên
Viết thuê đề tài giá rẻ trọn gói - KB Zalo/Tele : 0973.287.149
Luanvanmaster.com – Cần Kham Thảo - Kết bạn Zalo/Tele : 0973.287.149
3
Mục lục
Danh sách kí hiệu 4
Mở đầu 5
Chương 1. Phương trình Diophantine tuyến tính 7
1.1 Phương trình bậc nhất hai ẩn . . . . . . . . . . . . . . . . . . . . . . . 8
1.2 Phương trình bậc nhất nhiều ẩn . . . . . . . . . . . . . . . . . . . . . . 15
Chương 2. Một số phương trình Diophantine phi tuyến 23
2.1 Phương trình Pell loại 1 . . . . . . . . . . . . . . . . . . . . . . . . . . 23
2.2 Phương trình Pell loại 2 . . . . . . . . . . . . . . . . . . . . . . . . . . 30
2.3 Phương trình Pythagoras . . . . . . . . . . . . . . . . . . . . . . . . . 38
Chương 3. Liên phân số và ứng dụng trong phương trình Diophantine 45
3.1 Liên phân số hữu hạn . . . . . . . . . . . . . . . . . . . . . . . . . . . 45
3.2 Liên phân số vô hạn . . . . . . . . . . . . . . . . . . . . . . . . . . . . 49
3.3 Liên phân số vô hạn tuần hoàn . . . . . . . . . . . . . . . . . . . . . . 50
3.4 Áp dụng vào phương trình Diophante . . . . . . . . . . . . . . . . . . 56
3.4.1 Phương trình bậc nhất hai ẩn Ax + By = C . . . . . . . . . . . . 56
3.4.2 Phương trình x
2
dy
2
= 1 . . . . . . . . . . . . . . . . . . . 57
Kết luận 62
Tài liệu tham khảo 63
Viết thuê đề tài giá rẻ trọn gói - KB Zalo/Tele : 0973.287.149
Luanvanmaster.com – Cần Kham Thảo - Kết bạn Zalo/Tele : 0973.287.149
4
Danh sách kí hiệu
N
Z
Q
R
C
Fp
K[X]
dxe
deg P(X)
mod p
gcd(P(X); Q(X))
tập hợp các số tự nhiên
vành các số nguyên
trường các số hữu tỷ
trường các số thực
trường các số phức
trường có p phần tử
vành đa thức với hệ số trên trường K
trần của số x
bậc của đa thức P(X)
modulo p
ước chung lớn nhất của hai đa thức P(X) và Q(X)
Viết thuê đề tài giá rẻ trọn gói - KB Zalo/Tele : 0973.287.149
Luanvanmaster.com – Cần Kham Thảo - Kết bạn Zalo/Tele : 0973.287.149
5
Mở đầu
Phương trình Diophantine là một chủ đề lớn của Lý thuyết số, chứa đựng nhiều lý
thuyết toán học sâu sắc, gắn liền với nhiều tên tuổi của nhiều nhà toán học xuất sắc.
Mục tiêu của đề tài luận văn là: Tìm hiểu một số lớp phương trình Diophantine như:
phương trình Diophantine tuyến tính; một số phương trình Diophantine phi tuyến
(phương trình Pell, phương trình Pell mở rộng, phương trình Pythagoras Fermat).
Liên phân số và ứng dụng trong phương trình Diophantine. Về mặt ứng dụng, luận
văn sẽ áp dụng lý thuyết để soi sáng những bài toán số học ở phổ thông, hệ thống
hóa, tổng quát hóa và sáng tác ra những bài toán số học mới.
Luận văn sẽ cố gắng trở thành một tài liệu tham khảo tốt, thiết thực phục
vụ cho việc giảng dạy, nhất là việc giảng dạy và bồi dưỡng học sinh giỏi.
Ngoài ra thông qua việc viết luận văn, tác giả luận văn có cơ hội mở rộng
nâng cao hiểu biết về toán sơ cấp nói chung và số học nói riêng, hình thành
các kỹ năng chứng minh các định lí số học và giải các bài toán số học, phục
vụ tốt cho việc giảng dạy môn Toán ở trường phổ thông.
Nội dung của luận văn được trình bày trong ba chương như sau:
Chương 1. Phương trình Diophantine tuyến tính. Trong chương này
chúng tôi trình bày về phương trình bậc nhất hai ẩn, nhiều ẩn, và
một số bài toán chọn lọc.
Chương 2. Một số phương trình Diophantine phi tuyến. Trong chương
này chúng tôi trình bày nội dung chính về các phương trình Pell loại 1,
phương trình Pell loại , và phương trình Pythagoras.
Chương 3. Liên phân số và ứng dụng trong phương trình Diophantine. Trong
Viết thuê đề tài giá rẻ trọn gói - KB Zalo/Tele : 0973.287.149
Luanvanmaster.com – Cần Kham Thảo - Kết bạn Zalo/Tele : 0973.287.149
6
chương này chúng tôi trình bày một cách ngắn gọn các sự kiện về liên
phân số, đặc biệt là các ứng dụng của chúng để giải phương trình Pell.
Luận văn này được thực hiện tại Trường Đại học Khoa học - Đại học
Thái Nguyên và hoàn thành với sự hướng dẫn của GS.TSKH. Đặng Hùng
Thắng (Trường ĐHKHTN - ĐHQG Hà Nội). Tác giả xin được bày tỏ lòng
biết ơn chân thành và sâu sắc tới người hướng dẫn khoa học của mình,
người đã đặt vấn đề nghiên cứu, dành nhiều thời gian hướng dẫn và tận
tình giải đáp những thắc mắc của tác giả trong suốt quá trình làm luận văn.
Tác giả xin trân trọng cảm ơn Ban Giám hiệu Trường Đại học Khoa học - Đại
học Thái Nguyên, Ban Chủ nhiệm Khoa Toán–Tin, cùng các giảng viên đã tham
gia giảng dạy, đã tạo mọi điều kiện tốt nhất để tác giả học tập và nghiên cứu.
Tác giả muốn gửi những lời cảm ơn tốt đẹp nhất tới tập thể lớp Cao
học Toán khóa 9 (2015-2017) đã động viên và giúp đỡ tác giả rất nhiều
trong suốt quá trình học tập.
Nhân dịp này, tác giả cũng xin chân thành cảm ơn Sở Giáo dục và Đào tạo
Hải Phòng, Ban Giám hiệu và các đồng nghiệp ở Trường THPT Thái Phiên đã tạo
điều kiện cho tác giả hoàn thành tốt nhiệm vụ học tập và công tác của mình.
Cuối cùng, tác giả muốn dành những lời cảm ơn đặc biệt nhất đến đại gia đình vì
những động viên và chia sẻ những khó khăn để tác giả hoàn thành luận văn này.
Thái Nguyên, ngày 10 tháng 11 năm 2017
Tác giả
Bùi Hữu Mên
Viết thuê đề tài giá rẻ trọn gói - KB Zalo/Tele : 0973.287.149
Luanvanmaster.com – Cần Kham Thảo - Kết bạn Zalo/Tele : 0973.287.149
7
Chương 1
Phương trình Diophantine tuyến tính
Phương trình Diophantine là một trong những chủ đề sâu sắc và rất rộng
của Lý thuyết số. Mục đích của chương này là nghiên cứu về phương
trình Diophantine bậc nhất hai và nhiều ẩn. Như một minh họa cho lý
thuyết, các ví dụ là các bài toán trích từ các đề thi sẽ được trình bày.
Đặc tính của các phương trình Diophantine là chúng có một hay
nhiều ẩn số mà mọi hệ số đều là số nguyên và chỉ yêu cầu tìm các
nghiệm nguyên (hoặc nguyên dương). Nhà toán học nổi tiếng thời cổ đại
Diophantine đã có công lớn vì những nghiên cứu tiên phong về chúng.
Với một phương trình Diophantine cho trước ta có thể đặt ra các câu
hỏi sau đây (xếp theo thứ tự từ dễ đến khó):
Câu hỏi 1. Nó có nghiệm nguyên hay không ?
Câu hỏi 2. Nó có một số hữu hạn nghiệm hay có vô số nghiệm?
Câu hỏi 3. Hãy tìm tất cả các nghiệm của nó.
Chẳng hạn, ta hãy xét phương trình Diophantine
x
n
+ y
n
= z
n
trong đó n là số nguyên dương lớn hơn hay bằng 2. Với n = 2 phương trình trên
có vô số nghiệm và ta có thể tìm được tường minh tất cả các nghiệm của nó. Với
n > 2, nhà toán học thiên tài của thế kỷ 17 Pierre de Fermat khẳng định rằng
Viết thuê đề tài giá rẻ trọn gói - KB Zalo/Tele : 0973.287.149
Luanvanmaster.com – Cần Kham Thảo - Kết bạn Zalo/Tele : 0973.287.149
8
phương trình trên không có nghiệm nguyên dương. Kết luận này ngày
nay được mang tên là Định lí lớn Fermat hay Định lí cuối cùng của
Fermat. Người ta đã không tìm thấy dấu vết của chứng minh khẳng định
trên của Fermat mà chỉ thấy một ghi chú của Fermat bên lề cuốn sách
“Số học” của Diophantine: “Tôi đã tìm được một chứng minh thật là tuyệt
vời nhưng vì lề sách ở đây quá hẹp nên không thể viết ra”.
Năm 1983, nhà toán học 29 tuổi người Đức là Faltings đã chứng minh
thành công một giả thuyết của Mordell trong lĩnh vực Hình học đại số rồi từ đó
suy ra rằng phương trình x
n
+ y
n
= z
n
với n > 2 chỉ có một số hữu hạn nghiệm
nguyên. Với thành tựu này Faltings đã nhận được Giải thưởng Fields (giải
thưởng quốc tế cao nhất dành cho các nhà toán học không quá 40 tuổi).
Năm 1993 nhà toán học người Anh là Andrew Wiles đã công bố phép
chứng minh của Định lí lớn Fermat. Đây là một câu chuyện lớn của Toán
học, có thể tham khảo trong Amir D. Aczel [1].
Với sự ra đời của máy tính, người ta cũng đặt câu hỏi: Có tồn tại chăng
một thuật toán để với mọi phương trình Diophantine cho trước nhờ đó có thể
khẳng định được rằng phương trình này có nghiệm nguyên hay không. Tiếc
thay câu trả lời lại là: không có một thuật toán như vậy (Định lí Machiakevich).
1.1 Phương trình bậc nhất hai ẩn
Phương trình Diophantine đơn giản nhất là phương trình bậc nhất hai ẩn
ax + by = c (1.1)
trong đó a, b, c là những số nguyên cho trước khác 0. Vấn đề đặt ra là
với điều kiện nào của a, b, c thì phương trình (1.1) có nghiệm và nếu có
thì cách tìm nghiệm thế nào.
Định lí 1.1.1. Điều kiện cần và đủ để phương trình (1.1) có nghiệm nguyên là
Viết thuê đề tài giá rẻ trọn gói - KB Zalo/Tele : 0973.287.149
Luanvanmaster.com – Cần Kham Thảo - Kết bạn Zalo/Tele : 0973.287.149
9
(a; b) là ước của c.
Chứng minh. Điều kiện cần. Giả sử (x0; y0) là một nghiệm nguyên của
(1.1). Khi đó ax0 + by0 = c. Nếu d = (a; b) thì rõ ràng d j c.
Điều kiện đủ. Giả sử d = (a; b) và d j c. Ta có a = da1, b = db1, c = dc1
trong đó (a1; b1) = 1. Phương trình (1.1) tương đương với a1x + b1y = c1.
Xét a1 số fb1kg với k = 0; 1; 2; : : : ; a1 1. Vì (a1; b1) = 1 nên các số
này khi chia cho a1 sẽ cho ta các số dư khác nhau. Vậy tại k0, 0 k0 a1 1
sao cho b1k0 = c1 (mod a1). Điều này có nghĩa là:
c1 b1k0 = a1l0 với l0 2 Z hay c1 = a1l0 + b1k0:
Vậy (l0; k0) là một nghiệm của phương trình (1.1). Phép chứng minh
định lí được hoàn thành.
Tiếp theo ta hãy đi tìm tất cả các nghiệm của phương trình (1.1)
Định lí 1.1.2. Nếu (x0; y0) là một nghiệm nguyên của (1.1) thì nó có vô số
nghiệm nguyên và nghiệm nguyên (x; y) của nó được cho bởi công thức
8
x= x0 +
b
t;
d (1.2)
>
y= y 0
a t;
<
d
>
:
trong đó t 2 Z và d = (a; b):
Chứng minh. Trước hết ta kiểm tra mọi cặp số (x; y) cho bởi công thức
(1.2) là nghiệm. Thật vậy
ax + by = ax0 + by0 = c:
Đảo lại, giả sử (x1; y1) là một nghiệm của (1.1) tức là ax1 + by1 = c. Trừ
đẳng thức này vào đẳng thức ax0 + by0 = c ta thu được
a(x1 x0) = b(y0 y1): (1.3)
Viết thuê đề tài giá rẻ trọn gói - KB Zalo/Tele : 0973.287.149
Luanvanmaster.com – Cần Kham Thảo - Kết bạn Zalo/Tele : 0973.287.149
10
Vì d = (a; b) nên a = a1d, b = b1d với (a1; b1) = 1. Thay vào (1.3) ta được a1(x1
x0) = b1(y0 y1). Vì (a1; b1) = 1 nên y0 y1 = ta1 và x1 x0 = tb1. Vậy
y1 = y0 ta1 = y0 at và x1 = x0 +tb1 = x0 +bt :
d d
Phép chứng minh được kết thúc.
Thuật toán tìm nghiệm của phương trình Diophantine bậc nhất. Từ Định lí
1.1.2 ta thấy rằng để tìm tất cả các nghiệm của (1.1) ta chỉ cần tìm một nghiệm
(x0; y0) nào đó của nó. Ta gọi một nghiệm cụ thể như thế là một nghiệm riêng
còn công thức (1.2) được gọi là nghiệm tổng quát. Sau đây ta sẽ trình bày một
thuật toán cho phép xác định khá nhanh một nghiệm riêng của (1.1).
Giả sử q0; q1; : : : là một dãy các số nguyên dương. Với mỗi i 0
ta kí hiệu
[q0; q1; : : : ; qi] là phân số sau đây
1
[q0; q1; : : : ; qi] = q0 + :
1
q1 +
1
q2 + +
1
q
i 1 +q
i
Bằng phương pháp quy nạp có thể dễ dàng chứng minh được bổ đề sau:
Bổ đề 1.1.3. Giả sử fhng, fkng là hai dãy số nguyên được xác định như sau:
h 2 = 0; h 1 = 1; h1 = qihi 1 + hi 2; i 0;
k0 = 1; k1 = q1; ki = qihki 1 + ki 2; i 2:
Khi đó với mọi i 1 ta có:
(a) hiki 1 hi 1ki = ( 1)
i 1
;
(b) [q0; q1 ; : : : ; qi] =
h
i
.
k
i
Viết thuê đề tài giá rẻ trọn gói - KB Zalo/Tele : 0973.287.149
Luanvanmaster.com – Cần Kham Thảo - Kết bạn Zalo/Tele : 0973.287.149
11
Bây giờ cho hai số dương a, b với a > b. Ta hãy viết thuật toán Euclid
tìm ước chung lớn nhất của a và b.
a = bq0 + r1
b = r1q1 + r2
: : : (1.4)
r
n 1
= r
n
q
n 1
+ r
n+1
r
n
= r
n+1
q
n
:
Từ hệ (1.4) ta suy ra
a
= [q0; q1; : : : ; qn]
b
a
=
h
n :
b kn
Từ (a) ta suy ra (hn; kn) = 1. Do đó nếu (a; b) = 1 thì a = hn và b = kn. Vậy thì
akn 1 bhn 1 = ( 1)
n 1
:
Thành thử tồn tại x0 2 fkn 1g và y0 2 fhn 1g sao cho
ax0 + by0 = 1:
Ta thử từng trường hợp (nhiều nhất là bốn phép thử) để xác định x0, y0.
Như vậy để giải phương trình (1.1) ta sẽ tiến hành lần lượt các bước sau đây.
Bước 1. Tìm d = (a; b) sau đó chia hai vế cho d để được một phương
trình tương đương a1x + b1y = c1 , ở đó
a = da1; b = db1; c = dc1; (a1; b1) = 1:
Bước 2. Viết thuật toán Euclid cho hai số ja1j và jb1j. Giả sử ja1j > jb1j.
ja1j = jb1jq0 + r1
Viết thuê đề tài giá rẻ trọn gói - KB Zalo/Tele : 0973.287.149
Luanvanmaster.com – Cần Kham Thảo - Kết bạn Zalo/Tele : 0973.287.149
12
jb1j = r1q1 + r2
: : :
r
n 1
= r
n
q
n 1
+ r
n+1
r
n
= r
n+1
q
n
:
Bước 3. Tính [q0; q1; : : : ; qn 1] =
h
.
k
Bước 4. Lấy nghiệm riêng (x0
0
; y0
0
) của phương trình a1x + b1y = 1 thoả mãn điều
kiện jx0
0
j = k, jy0
0
j = h.
Ta xác định dấu của x0
0
và y0
0
bằng cách thử.
Bước 5. Ta có x0 = c1x0
0
, y0 = c1y0
0
là nghiệm riêng của phương trình (1.1). Khi
đó nghiệm tổng quát là
x = x0 + b1t; y = y0 + a1t; với t 2 Z:
Ví dụ 1.1.4. Giải phương trình Diophantine 342x 123y = 15:
Lời giải. Ta sẽ làm lần lượt theo các bước như trên.
Bước 1. Ứớc chung lớn nhất của 342 và 123 là 3. phương trình đã cho tương đương
với 114x 41y = 5.
Bước 2. Ta viết thuật toán Euclid cho 114 và 41.
114= 41 2+32
41= 32 1+9
32=9 3+5
9=5 1+4
5=4 1+1
4=1 4:
Viết thuê đề tài giá rẻ trọn gói - KB Zalo/Tele : 0973.287.149
Luanvanmaster.com – Cần Kham Thảo - Kết bạn Zalo/Tele : 0973.287.149
Bước 3. Ta biểu diễn theo liên phân số
h
= 2 +
Như vậy h = 25, k = 9.
13
1
= 25:
19
1+
1
3 +
1
1 +
1
Bước 4. Lấy nghiệm riêng x0
0
, y0
0
của phương trình 114x 41y = 1 thoả mãn điều
j 0j j 0j 0 0
kiện x
0
= 9 và y
0
= 25: Bằng cách thử ta tìm được x
0
= 9, y
0
= 25.
Bước 5. Nghiệm riêng x0, y0 của phương trình đã cho là x0 = 9 5 = 45, y0 =
25 5 = 125 và nghiệm tổng quát là
x = 45 + 41t; y = 125 + 114t; với t 2 Z:
Nhận xét 1.1.5.
1. Nếu a j c ta có thể lấy nghiệm riêng
x
0 =c ; y0 = 0:
a
2. Xét trường hợp (a; b) = 1. Theo Định lí Euler ta có
a
j(b)
1 = kb với k 2 Z:
Vậy ca
j(b)
bkc = c. Do đó
x0 = ca
j(b)
1; y0 = kc
là một nghiệm riêng của phương trình đang xét.
Viết thuê đề tài giá rẻ trọn gói - KB Zalo/Tele : 0973.287.149
Luanvanmaster.com – Cần Kham Thảo - Kết bạn Zalo/Tele : 0973.287.149
14
Một số bài toán chọn lọc
Bài toán 1.1.6. Cho hai số nguyên dương a, b. Chứng minh rằng (a; b) =
1 khi và chỉ khi tồn tại các số nguyên dương u, v sao cho au bv = 1:
Lời giải. Điều kiện đủ là hiển nhiên.
Đảo lại, giả sử (a; b) = 1. Theo Định lí 1.1.1, tồn tại các số nguyên x0,
y0 để cho ax0 + by0 = 1. Đặt
8
>u = x0 + bt;
trong đó t
<
v= at y0;
2
Z >
:
x0 y0
t > ; t > :
b b
Khi đó u và v là các số nguyên dương và au bv = ax0 + by0 = 1.
Bài toán 1.1.7. Giả sử (l; m) = 1 và a
l
= b
m
, trong đó a; b; l; m 2 N . Khi
đó tồn tại n để a = nm
, b = nl
:
Lời giải. Theo Bài toán 1.1.6 tồn tại các số r; s 2 N để lr ms = 1. Ta có
alr bmr br m
a = alr ms
= :
= =
ams ams a
s
p b
r p b
r
2 N .
m m
Suy ra a = a
s
. Vì a là một số hữu tỉ nên nó phải là số nguyên. Vậy n = as
Suy ra a = n
m
. Từ đó b
m
= n
l
= n
ml
, nên b = n
l
. Bài toán được chứng minh xong.
Bài toán 1.1.8. Cho a 2 N . Hãy tìm g = (am
1; an
1):
Lời giải. Trước hết xét trường hợp (m; n) = 1: Vì
(a 1) j am
1; (a 1)an
1
nên a 1 là ước của g.
Đảo lại, ta sẽ chứng minh g cũng là ước của a 1. Theo Bài toán 1.1.6
tồn tại các số u; v 2 N sao cho mu nv = 1:
Viết thuê đề tài giá rẻ trọn gói - KB Zalo/Tele : 0973.287.149
Luanvanmaster.com – Cần Kham Thảo - Kết bạn Zalo/Tele : 0973.287.149
15
Vì g j a
m
1, g j a
n
1 nên cũng có g j a
mu
1, g j a
nv
1. Suy ra
g j amu
anv = anv(amu nv 1) = anv
(a 1):
Mặt khác, dễ thấy (g; a) = 1. Vậy g j (a 1):
Như vậy nếu (m; n) = 1 thì (a
m
1; a
n
1) = a 1.
Với m, n bất kì, giả sử d = (m; n): Khi ấy m = dm1, n = dn1 và (m1; n1) = 1:
Ta có
(am
1; an
1) = ((ad
)m
1 1; (ad
)n
1 1) = ad
1:
Tóm lại ta có công thức
(a
m
1; a
n
1) = a(m; n) 1:
Bài toán 1.1.9. Cho (a; b) = 1 trong đó a; b 2 N . Tìm giá trị c 2 N lớn
nhất để phương trình ax + by = c không có nghiệm nguyên dương.
Lời giải. Ta chứng minh rằng c = ab là giá trị cần tìm. Giả sử c > ab. Xét
b số a; 2a; : : : ; ba. Vì (a; b) = 1 nên các số này khi chia cho b sẽ cho
các số dư khác nhau. Vậy tồn tại số k, 1 k b, sao cho ka c (mod b). Suy
ra c ka = lb trong đó l 2 Z. Nếu c > ab thì c > ka, do đó lb > 0 hay l 2 N .
Như vậy (k; l) là nghiệm nguyên dương của phương trình ax + by = c:
Mặt khác, giả sử (x0; y0) là nghiệm nguyên dương của phương trình
ax + by = ab. Khi đó ax0 = ab by0 = b(a y0).
Vì (a; b) = 1 nên từ đó suy ra x0
..
. b. Tương tự, y0
..
. a. Như vậy x0 b,
y0 a, do đó ab = ax0 + by0 2ab. Vô lý. Phép chứng minh được kết thúc.
1.2 Phương trình bậc nhất nhiều ẩn
Trong mục này ta mở rộng kết quả của mục trước bằng cách xét
phương trình Diophantine bậc nhất n ẩn
a1x1 + a2x2 + : : : + anxn = c: (1.5)
Viết thuê đề tài giá rẻ trọn gói - KB Zalo/Tele : 0973.287.149
Luanvanmaster.com – Cần Kham Thảo - Kết bạn Zalo/Tele : 0973.287.149
16
trong đó a1; a2; : : : ; an và c là các số nguyên cho trước, n 2.
Đối với bất kỳ một phương trình nào, câu hỏi đầu tiên là, trong những tình huống
nào của hệ số, ta có thể khẳng định về tính tồn tại nghiệm của nó. Về sự tồn tại
nghiệm của phương trình Diophantine bậc nhất n ẩn này ta có định lí sau đây.
Định lí 1.2.1. Điều kiện cần và đủ để phương trình (1.5) có nghiệm là
(a1; a2; : : : ; an) j c:
Chứng minh. Điều kiện cần là hiển nhiên. Ta sẽ chứng minh điều kiện đủ
bằng phương pháp quy nạp, với n = 1 điều khẳng định là đúng do Định lí
1.1.1. Giả sử định lí đúng với n = 1, ta chứng minh nó đúng với n. Kí hiệu
ai c
bi = ; c1 = ; ở đó d = (a1 ; a2; : : : ; an):
d d
Phương trình (1.5) tương đương với
b1x1+ b2x2 + : : : + bnxn = c1: (1.6)
trong đó (b1; b2; : : : ; bn) = 1. Đặt b = (b1; b2; : : : ; bn 1). Ta có (b; bn) =
1. Theo Định lí 1.1.1 tồn tại số nguyên ln và k sao cho:
bnln + bk = c1: (1.7)
Kí hiệu b
0
=bi với i = 1; 2; : : : ; n 1. Ta có (b
0
; b
0
; : : : s; b
0
1
)=1:
b
i 1 2 n
Theo giả thiết quy nạp tồn tại các số nguyên l1; l2; : : : ; ln 1 sao cho
b1
0
l1 + b2
0
l2 + : : : + bn
0 1
l
n 1
= k
hay
b1l1 + b2l2 + : : : + bn 1
l
n 1 = bk: (1.8)
Từ (1.7) và (1.8) ta suy ra
b1l1 + b2l2 + : : : + bn 1ln 1 + bnln = c1
tức là (l1; l2; : : : ; ln) là nghiệm của phương trình (1.6). Định lí được chứng minh.
Viết thuê đề tài giá rẻ trọn gói - KB Zalo/Tele : 0973.287.149
Luanvanmaster.com – Cần Kham Thảo - Kết bạn Zalo/Tele : 0973.287.149
17
Chúng ta không đi sâu vào việc tìm biểu thức cho nghiệm tổng quát của nó
như đã làm đối với trường hợp n = 2. Tuy nhiên có thể thấy rằng nếu phương
trình (1.5) có nghiệm nguyên a1; a2; : : : ; an thì nó sẽ có vô số nghiệm nguyên
phụ thuộc vào n 1 tham số. Thật vậy, dễ dàng kiểm tra được tất cả các bộ n số
nguyên x1; x2; : : : ; xn xác định như sau là nghiệm của (1.5),
8
x1 = a1 + ant1
>x2 + a2 + ant2
>
>
>
>
>
>
: : :
>
>
>
>
<
>
xn 1 = an 1 + antn 1
>
>
>
x= a
n
at
1
a t : : : a
n 1
t
n 1
> n 1 2 2
>
>
>
>
>
,
> được chọn tuỳ ý.
t 2 Z i =
1
; 2; : : : n 1
trong đó i :
Bây giờ, ta sẽ thảo luận về cách giải phương trình (1.5). Về mặt thực
hành ta có thể tiến hành theo hai cách sau đây.
Cách 1. Đưa (1.5) về trường hợp có một hệ số bằng 1.
Cách 2. Nếu phương trình (1.5) có hai hệ số nguyên tố cùng nhau,
chẳng hạn (a1; a2) = 1 thì ta viết dưới dạng
a1x1 + a2x2 = c a3x3 : : : anxn
rồi giải phương trình theo hai ẩn x1, x2.
Ta xét hai ví dụ sau đây, nó sẽ lần lượt minh họa cho hai cách trên.
Ví dụ 1.2.2. Tìm tất cả các nghiệm nguyên của phương trình
6x + 45y + 6z 10t = 13:
Viết thuê đề tài giá rẻ trọn gói - KB Zalo/Tele : 0973.287.149
Luanvanmaster.com – Cần Kham Thảo - Kết bạn Zalo/Tele : 0973.287.149
Lời giải. Phương trình đã cho được viết dưới dạng
6(x + z) + 10(4y t) + 5y = 13:
Viết thuê đề tài giá rẻ trọn gói - KB Zalo/Tele : 0973.287.149
Luanvanmaster.com – Cần Kham Thảo - Kết bạn Zalo/Tele : 0973.287.149
18
Đặt
x + z = x1; 4y t = x2
ta được 6x1 + 10x2 + 5y = 13. Suy ra
x1 + 10x2 + 5(y + x1) = 13:
Đặt x1 + y = x3 ta được x1 + 10x2 + 5x3 = 13: Vậy
x1 = 13 10x2 5x3:
Từ đây, bằng tính toán trực tiếp ta có
y = x3 x1 = x3 (13 10x2 5x3) = 10x2 + 6x3 13;
t = 4y x2 = 39x2 + 24x3 52;
x = x1 z = 13 10x2 5x3 z:
Vậy nghiệm tổng quát của phương trình đã cho là
x = 13 10x2 5x3 x4;
y = 10x2 + 6x3 13;
z = x4;
t = 39x2 + 24x3 52:
trong đó x2, x3, x4 là các số nguyên tuỳ ý.
Ví dụ 1.2.3. Giải phương trình
6x + 15y + 10z = 3: (1.9)
Lời giải. Ta có thể viết (1.9) dưới dạng
6(x + z) + 15y = 3 4z:
Đặt u = x + z ta có 15y + 4z = 3 6u. Ta thấy ( 1; 4) là nghiệm riêng của
15y + 4z = 1. Do đó ( 3 + 6u; 12 24u) là nghiệm riêng của 15y + 4z = 3
6u. Suy ra nghiệm tổng quát của nó là
y = 3 + 6u + 4t; z = 12 24u 15t:
Viết thuê đề tài giá rẻ trọn gói - KB Zalo/Tele : 0973.287.149
Luanvanmaster.com – Cần Kham Thảo - Kết bạn Zalo/Tele : 0973.287.149
19
Từ u = x + z suy ra
x = u z = u (12 24u 15t) = 12 + 25u + 15t:
Vậy nghiệm tổng quát của (1.9) là
8
>x = 12 + 25u + 15t;
>
>
>
<
y = 3 + 6u + 4t;
>
>
>
>z = 12 24u 15;
:
với u; t 2 Z.
Một số bài toán chọn lọc
Bài toán 1.2.4 (Đề thi Vô định Toán Quốc tế 1983). Cho a, b, c là các số
nguyên đôi một nguyên tố cùng nhau. Chứng minh rằng 2abc ab bc ca là
số nguyên lớn nhất không viết được dưới dạng xbc + yca + zab với x, y,
z là những số không âm.
Lời giải. Bài toán tương đương với việc chứng minh rằng
Khẳng định 1. phương trình xbc + yca + zab = 2abcab bc ca không có
nghiệm nguyên không âm;
Khẳng định 2. Nếu n > 2b ab bc ca thì phương trình có nghiệm nguyên
không âm.
Chứng minh Khẳng định 1. Giả sử tồn tại x0; y0; z0 2 N sao cho
x0bc + y0ca + z0ab = 2abc ab bc ca:
Điều này tương đương với
bc(x0 + 1) + ca(y0 + 1) + ab(z0 + 1) = 2abc:
Viết thuê đề tài giá rẻ trọn gói - KB Zalo/Tele : 0973.287.149
Luanvanmaster.com – Cần Kham Thảo - Kết bạn Zalo/Tele : 0973.287.149
20
Suy ra ab(z0 + 1)
..
. c. Vì (ab; c) = 1 nên z0 + 1
..
. c. Mặt khác z0 + 1 2
N nên z0 + 1 c: Tương tự y0 + 1 b, x0 + 1 a. Vậy
bc(x0 + 1) + ca(y0 + 1) + ab(z0 + 1) 3abc:
Điều này vô lí.
Chứng minh Khẳng định 2. Xét ab có dạng kbc + lac, với 0k a 1, 0 l
b 1:.
Các số này khi chia cho ab sẽ cho ta các số dư khác nhau. Thật vậy, giả sử
k1bc + l1ac k2bc + l2ac (mod ab):
. .
Vì (ab; c) = 1 nên suy ra b(k1 k2) a(l1
.
k2)
.
l2) . ab: Từ đó b(k1 . a và a(l1
. . .
.
k
2
.
l
2
.
k2j < a, jl1 l2j < b
l2) . b. Vì (a; b) = 1 nên k1 . a và l1 . b. Mặt khác jk1
nên l1 l2 = 0, k1 k2 = 0. Do đó tồn tại x0; y0 2 Z
+
, 0 x0 a 1, 0y0 b 1
để cho
bcx0 + acy0 n (mod ab):
Vậy tồn tại z0 2 Z để bcx0 + acy0 + abz0 = n: Ta có
ab(z0 + 1) = n bcx0 acy0 + ab
> 2abc bcx0 acy0 =
bc(a 1 x0) + ac(b
Do đó z0 + 1 > 0. Suy ra z0 0.
bc ca
1 y0) 0:
Bài toán 1.2.5. Cho các số nguyên không âm a, b thoả mãn điều kiện 5a
7b:
Chứng minh rằng hệ
8
<x + 2y + 3z + 7u = a;
:y + 2z + 5u = b
luôn có nghiệm nguyên không âm.
Viết thuê đề tài giá rẻ trọn gói - KB Zalo/Tele : 0973.287.149
Luanvanmaster.com – Cần Kham Thảo - Kết bạn Zalo/Tele : 0973.287.149
21
Chứng minh. Giả sử b = 5u + v với 0 u, 0 v 4. Từ điều kiện a
7
5
b
suy ra
a
7(5u+v)
. Từ đây ta có
5
a 7u
7v
:
5
Như vậy hệ phương trình có thể viết là
n
x + 2y + 3z = a yu y + 2z = b 5u = v:
Với mỗi 0 v 4 ta sẽ chọn y, z thích hợp để có x = z 7u 2y
3z 0.
(1.10)
(1.11)
Nếu v = 0, ta lấy y = z = 0. Khi đó x = a 7u 7
5
v
= 0 (theo (1.10))).
Nếu v = 1, ta thấy y = 1, z = 0. Khi đó
x = a yu 2
7v
2 =
7
2> 1:
5 5
Vậy x 0:
Nếu v = 2, ta lấy y = 0, z = 1. Khi đó
x = a 7u 3
7v
3 =
14
3> 1:
5 5
Vậy x 0:
Nếu v = 3, ta lấy y = z = 1. Khi đó
x = a 7u 5
7v
5 =
21
5> 1:
5 5
Vậy x 0:
Nếu v = 4, ta lấy y = 0, z = 2. Khi đó
x = a 7u 6
7v
6 =
28
6> 1:
5 5
Vậy x 0:
Tóm lại với mọi v = 0; 1; 2; 3; 4 ta đều chọn được y, z, x không âm để
(1.11) được nghiệm đúng. Bài toán được giải xong.
Viết thuê đề tài giá rẻ trọn gói - KB Zalo/Tele : 0973.287.149
Luanvanmaster.com – Cần Kham Thảo - Kết bạn Zalo/Tele : 0973.287.149
22
Bài toán 1.2.6 (Đề thi vô địch Mỹ 1982). Chứng minh rằng tồn tại số tự
nhiên k sao cho tất cả các số k
n
+ 1 với n = 1; 2; : : : đều là hợp số.
Lời giải. Xét các số Fermat Fm = 22m
+ 1:
Ta đã biết Fm là số nguyên tố với m = 0; 1; 2; 3; 4, trong khi Euler phát hiệu
rằng F5 là một hợp số, F5 là tích của hai số nguyên tố 641 và 6700417.
Ký hiệu
ai = Fi với i = 0; 1; 2; 3;
4;
a5 = 641a6 = 6700417:
Theo Định lí Trung Hoa về thặng dư, tồn tại số tự nhiên k > max(a0; : : : ; a6) để
k 1 (mod am) với m = 0; 1; 2; : : : ; 5 và k 1 (mod a6).
Ta sẽ chứng minh k chính là số cần tìm. Xét n bất kỳ, n có thể viết
dưới dạng n = 2
m
p với 0 m và p lẻ.
(a) Nếu m 4, ta có k 2
n
+ 1 2n + 1 (mod am).
Mặt khác 2n + 1 = 2
2m
p + 1 = (2
2m
)p + 1
..
. 2
2m
+ 1 = a
m
: Do đó k
2
n
+ 1
..
. a
m
. Vì k 2
n
+ 1 > k > a
m
nên k 2
n
+ 1 là hợp số.
(b) Nếu m = 5 thì k 2
n
+ 1 2n + 1 (mod a5).
Mặt khác 2n
+1 = 225 p
+1
..
. 225
+1 = F5
..
. 641 = a5. Vậy k 2n
+1
..
. a5. Mặt khác k 2n
+ 1 > a5, suy ra k 2n
+ 1 là hợp số.
(c) Nếu m 6, khi đó n = 26
b với b 2 Z. Ta có
k 2
n
+ 1 2
n
+ 1 (mod a6);
2n = 22
5 2b
= (F5 1)2b ( 1)2b
1 (mod a6) vì F5 .
..
a6:
Do đó k 2
n
+ 1 1 + 1 = 0 (mod a6). Vì k 2
n
+ 1 > a6 nên k 2
n
+ 1 là hợp
số.
Vậy với mọi n, số k 2n
+ 1 luôn là hợp số.
Viết thuê đề tài giá rẻ trọn gói - KB Zalo/Tele : 0973.287.149
Luanvanmaster.com – Cần Kham Thảo - Kết bạn Zalo/Tele : 0973.287.149
23
Chương 2
Một số phương trình
Diophantine phi tuyến
2.1 Phương trình Pell loại 1
Phương trình Pell loại 1 là phương trình có dạng
x
2
Dy
2
= 1; (2.1)
trong đó D 2 N và ta yêu cầu tìm nghiệm x; y 2 N . Trong tiết này khi nói
đến nghiệm của (2.1) ta hiểu là nghiệm nguyên dương.
Định lí 2.1.1 (Điều kiện tồn tại nghiệm). Phương trình (2.1) có nghiệm
nguyên dương khi và chỉ khi D là số không chính phương.
Chứng minh. Giả sử D = m
2
. Khi đó
x
2
Dy
2
= x
2
m
2
y
2
= 1 ! (x my)(x + my) = 1
! x my = x + my = 1 ! x = 1; y = 0:
Vậy (2.1) không có nghiệm nguyên dương.
Ngược lại giả sử D là số không chính phương. Ta có các bổ đề sau
Bổ đề 2.1.2. Cho a 2= Q. Khi đó tồn tại vô số cặp nguyên (h; k) với k > 0 sao cho
a k < k
1
2 :
h
Viết thuê đề tài giá rẻ trọn gói - KB Zalo/Tele : 0973.287.149
Luanvanmaster.com – Cần Kham Thảo - Kết bạn Zalo/Tele : 0973.287.149
24
Chứng minh. Ta sử dụng nhận xét đã biết sau: Với mỗi q 2 N tồn tại cặp
số nguyên (h; k) với 1 k q sao cho
a
Ký hiệu
h < 1 :
k kq
A = (h; k) : a k < k
1
2 :
h
Ta phải chứng minh
j
A
j j j
< ¥. Khi đó tồn tại e sao cho
= ¥. Giả sử trái lại A
a k > e với mọi (h; k) 2 A. Chọn q 2 N sao cho
h
1
< e: (2.2)
q
Theo nhận xét trên tồn tại cặp số nguyên (h0; k0) với 1 q sao cho
a k0 < k0q k0 : (2.3)
h0 1 12
Từ (2.3) ta có (h0; k0) 2 A ! h0 1 1 a h0 > e. Mâu
a k0 > e. Nhưng q k0q > k0
thuẫn với (2.2).
Bổ đề 2.1.3. Tồn tại vô số cặp số nguyên dương (x; y) sao cho
jx
2 Dy
2
j < 1 + 2
p
:
D
Chứng minh. Theo Bổ đề 2.1.2 tồn tại vô số cặp số nguyên (x; y) sao cho
0 < pD y < y
1
2 :
x
Suy ra y + pD = y p
D + 2p
D < y
1
2 + 2p
D:
x x
Vậy
y y + p D
jx2 Dy2j =jx ypDjjx + ypDj = y2 pD
x x
2 1 1 p 1 p p
< y ( +2 D)= + 2 D 1
+
2 D
y2 y2 y2
< :
Viết thuê đề tài giá rẻ trọn gói - KB Zalo/Tele : 0973.287.149
Luanvanmaster.com – Cần Kham Thảo - Kết bạn Zalo/Tele : 0973.287.149
25
Chứng minh định lý Từ bổ đề 2.1.3 (x; y) tồn tại vô số cặp số nguyên
dương (x; y) sao cho
jx
2 Dy
2
j < 1 + 2
p
:
D
p p
Đặt I = [ 1 2 d; 1 + 2 d]. Với mỗi k 2 I ký hiệu
Ak = f(x; y) 2 N : x
2
Dy
2
= kg
Do đó tồn tại k 2 I để jAkj = ¥. Suy ra tồn tại (x1; y1) 6= (x1; y1) 2 Ak để
x1 x2 (mod jkj) y1 y2 (mod jkj);
x1
2
Dy1
2
= x2
2
Dy2
2
= k;
Xét tích
(x1 y1 p )(x2 + y2 p )x1x2 Dy1y2 +
p
(x1y2 x2y1) (2.4)
D D D
Vì
x1x2 Dy1y2 x1
2
Dy1
2
0 (mod jkj)
x1y2 x2y1 x1y1 x1y1 0 (mod jkj):
Vậy tồn tại u; v 2 Z sao cho
x1x2 Dy1y2 = ku (2.5)
x1y2 Dy1x2 = kv (2.6)
Từ (2.4), (2.5), (2.6) suy ra
(x1 y1
p
)(x2 + y2) = k(u + v
p
);
D D
(x1 + y1)(x2 y2
p
) = k(u v
p
):
D D
Nhân hai đẳng thức trên với nhau và chú ý rằng (x1; y1; (x2; y2) 2 Ak ! x1
2
dy1
2
=
x2
2
dy2
2
= k ta được
k2 = k2(u2 Dv2) ! u2 Dv2 = 1:
Viết thuê đề tài giá rẻ trọn gói - KB Zalo/Tele : 0973.287.149
Luanvanmaster.com – Cần Kham Thảo - Kết bạn Zalo/Tele : 0973.287.149
26
Ta chứng minh u; v > 0. Rõ ràng u > 0. Nếu trái lại v = 0 thì u = 1 ! (x1
y1
p
)(x2 + y2
p
) = k = (x1
2
Dy1
2
) = (x1 y1
p
)(x1 + y1
p
) ! x2 +
D D D D
p p
y
2 D = x1 + y1 D ! x1 = x2; y1 = y2. Ta có mâu thuẫn. Vậy (u; v) là nghiệm
nguyên dương của phương trình (2.1).
Định lí 2.1.4 (Công thức nghiệm). Ký hiệu (a; b) là nghiệm nhỏ nhất của
phương trình
x2
Dy2
= 1:
Khi đó dãy (xn; yn) cho bởi
(a + b
p
)n
+ (a b
p
)
n
D D
xn = ;
2
(a + b
p
)
n
(a b
p
)
n
D D
yn = 2p :
D
cho tất cả các nghiệm của (2.2)
Dãy nghiệm (xn; yn) cũng có thể xác định theo công thức truy hồi sau
x0 = 1; x1 = a; xn+2 = 2axn+1 xn; (2.7)
y0 = 0; y1 = b; yn+2 = 2ayn+1 yn: (2.8)
Chứng minh. Ta có
xn + yn
p
= (a + b
p
)
n
; xn yn = (a bp )
n
: (2.9)
D d d
Suy ra (xn
2
Dy
2
n) = (a2 Db2)
n
= 1.
Đảo lại giả sử (x; y) là nghiệm bất kỳ của (2.1). Ta chứng minh tồn tại
n 2 N để x = xn; y = yn. Vì D không chính phương nên điều này tương
đương Tồn tại n 2 N để
x + y
p
D = xn + yn
p
D = (a + b
p
D)
n
:
Chứng minh bằng phản ứng. Giả sử trái lại
x + y
p
D 6= (a + b
p
D)
n
8n 2 N :
Viết thuê đề tài giá rẻ trọn gói - KB Zalo/Tele : 0973.287.149
Luanvanmaster.com – Cần Kham Thảo - Kết bạn Zalo/Tele : 0973.287.149
27
Khi đó tồn tại m 2 N sao cho
(a + b
p
D)
m
< x + y
p
D < (a + b
p
D)
m+1
:
Nhân hai vế với (a bp )
m
ta được
D
1 < (x + y
p
)(a b
p
)
m
< a + b
p
:
D D D
Do (2.9) ta có
(x + y
p
)(a b
p
)m
= (x + y
p
)(xm ym
p
)
D D D D
= (xxm Dyym) + (xmy ymx)
p
D
= u + v
p
;
D
ở đó u = xxm Dyym; v = xmy ymx. Vậy
1 < u + v
p
< a + b
p
: (2.10)
d d
Ta có
u2 Dv2 = (xxm Dyym)
2
D(xmy ymx)2
;
= (x
2
Dy
2
)(xm
2
Dy
2
m) = 1:
p p p p
Lại có x > y d; xm > ym d nên u > 0. Lại có (u v D)(u + v D) = 1 và 1 <
p p p
u + v d nên 0 < u v d < 1 < u + v d ! v > 0. Vậy (u; v) là nghiệm của (2.1)
p p
do đó a u; b u ! a + b d u + v d trái với (2.10). Định lý được chứng
minh.
Từ định lý trên ta thấy việc tìm nghiệm của phương trình Pell (2.1) quy về tìm
nghiệm nhỏ nhất (a; b) của nó. Cách đơn giản nhất là thử bằng tay: Thay lần lượt y =
1; 2; : : : vào biểu thức 1 + Dy
2
cho tới khi nào được số chính phương thì dừng lại. Vì
phương trình (2.1) có nghiệm nên chắc chắn quá trình này sẽ dừng lại sau
p
b phép thử. Khi đó nghiệm nhỏ nhất là (a; b) với a = 1 + Db
2
. Nếu
nghiệm nhỏ
Viết thuê đề tài giá rẻ trọn gói - KB Zalo/Tele : 0973.287.149
Luanvanmaster.com – Cần Kham Thảo - Kết bạn Zalo/Tele : 0973.287.149
28
nhất b này lớn thì cách thử này không khả thi. Thí dụ với phương trình
x
2
61y
2
thì cặp nghiệm nhỏ nhất là
a = 1766319049; b = 226153980:
Sau đây ta sẽ trình bày một thuật toán sử dụng liên phân số để tìm một
nghiệm nhỏ nhất (a; b) của (2.1).
Định nghĩa 2.1.5. Cho a0; a1; a2; : : : là dãy số nguyên trong đó ai > 0; i 1. Đặt
Ck = [a0; a1; : : : ; ak]:
Khi đó tồn tại giới hạn
limCk = a:
Ta gọi a là giá trị của liên phân số vô hạn [a0; a1; a2; : : :]và viết
a = [a0; a1; a2; : : :]:
Định lí 2.1.6. a = [a1; a1; a2; : : :] là một số vô tỷ. Ngược lại mỗi số vô tỷ
đều biểu diễn duy nhất dưới dạng một liên phân số vô hạn.
Định nghĩa 2.1.7. Ta gọi liên phân số vô hạn a = [a1; a1; a2; : : :] là tuần hoàn nếu
dãy (an) tuần hoàn kể từ một chỉ số nào đó tức là: Tồn tại các số nguyên
dương m và k sao cho mọi n m ta có an = am+k. Số nguyên dương k
được gọi là chu kỳ của liên phân số a = [a0; a1; a2; : : :]. Khi đó ta viết
a = [a0; a1; a2; : : : ; am 1; am; am+1; : : : ; am+k 1]:
p
Nếu D là số không chính phương, biểu diễn liên phân số của D được
cho bởi
p
Định lí 2.1.8. Biểu diễn liên phân số của D là tuần hoàn và có
dạng p
D = [a; a1; a2; : : : ; an; 2a]:
p
với a = [ D]. Hơn nữa có công thức tường minh để xác định dãy (a1; : : : ; an). Chú
ý rằng dãy (a1; : : : ; an) là đối xứng tức là
Viết thuê đề tài giá rẻ trọn gói - KB Zalo/Tele : 0973.287.149
Luanvanmaster.com – Cần Kham Thảo - Kết bạn Zalo/Tele : 0973.287.149
a1 = an; a2 = an 1; : : :
Viết thuê đề tài giá rẻ trọn gói - KB Zalo/Tele : 0973.287.149
Luanvanmaster.com – Cần Kham Thảo - Kết bạn Zalo/Tele : 0973.287.149
29
Ví dụ 2.1.9.
p
23 = [4;1;3;1;8]
p
29 = [5;2;1;1;2;10]
p
31 = [5;1;1;3;5;3;1;1;10]
p
46 = [6;1;2;1;1;2;6;2;1;1;2;1;12]
p
76 = [8;1;2;1;1;5;4;5;1;1;2;1;16]
p
97 = [9;1;5;1;1;1;1;1;1;5;1;18]:
Định lí 2.1.10. Cho phương trình Pell
x
2
Dy
2
= 1: (I)
1. Biểu diễn
p
thành liên phân số
D
p
D = [a; a1; a2; : : : ; an; 2a]:
2. Nếu chu kỳ n là số chẵn ta tính giản phân thứ n1
pn 1
Cn 1 = :
3. Khi đó (pn 1; qn 1) là nghiệm nhỏ nhất của (2.1)
4. Nếu chu kỳ n là số lẻ ta tính giản phân thứ 2n 1
C
1
=
p
2n 1
:
2n
q
2n 1
5. Khi đó (p2n 1; q2n 1) là nghiệm nhỏ nhất của (2.1)
Ví dụ 2.1.11. Tìm nghiệm nhỏ nhất của phương trình x
2
14y
2
= 1. Ta có
p
=
14
]. Chu kỳ n = 4 là số chẵn. Vậy ta tính giản phân
[3;1;2;1;6
1
C3 = [3;1;2;1] = 3+
1
1 +
1
2 +
1
Viết thuê đề tài giá rẻ trọn gói - KB Zalo/Tele : 0973.287.149
Luanvanmaster.com – Cần Kham Thảo - Kết bạn Zalo/Tele : 0973.287.149
30
= 15:
4
Vậy nghiệm nhỏ nhất là (15; 4).
p
Ví dụ 2.1.12. Tìm nghiệm nhỏ nhất của phương trình x2
13y2
= 1. Ta có
13 = [3;1; 1; 1; 1; 6] = [3; 1; 1; 1; 1; 6; 1; 1; 1; 1; 6; : : : Chu kỳ n = 5 là số
lẻ. Vậy ta tính giải phân
C9 = [3;1;1;1;1;6;1;1;1] = 3+ 1 =649 :
1
1 +
180
1+
..
. + 1
1
1 +
1
Vậy nghiệm nhỏ nhất là (649; 180) Trở lại phương trình x
2
61y
2
= 1. Ta có
p ]
76 = [7;1;4;3;1;2;2;1;3;4;1;14
Chu kỳ n = 11 là số lẻ. Ta tính giản phân
C21 = [7;1;4;3;1;2;2;1;3;4;1;14;1;4;3;1;2;2;1;3;4;1]
= 7 +
1
1
1 + 1
4 +
1
3+
..
. + 4 + 1
1
= 1766319049
226153980
Vậy nghiệm nhỏ nhất là (1766319049; 226153980)
2.2 Phương trình Pell loại 2
Phương trình Pell loại 2 là phương trình
x
2
Dy
2
= 1 (2.11)
ở đó D 2 N . Nghiệm của (2.11) luôn được hiểu là nghiệm nguyên dương.
Viết thuê đề tài giá rẻ trọn gói - KB Zalo/Tele : 0973.287.149
Luanvanmaster.com – Cần Kham Thảo - Kết bạn Zalo/Tele : 0973.287.149
31
Định lí 2.2.1. Điều kiện cần để (2.11) có nghiệm là D không là số chính
phương và D không có ước nguyên tố dạng 4k + 3.
Chứng minh. i) nếu D = m
2
thì (2.11) trở thành (my x)(my +x) = 1 ! my
x = my + x = 1 ! x = 0 vậy (2.11) vô nghiệm.
ii) Nếu D có ước nguyên tố p = 4k +3 : Giả sử (x; y) là nghiệm. Khi đó
x
2
+1 = Dy
2
! pjx
2
+ 1. Vì p = 4k + 3 nên pj1. Mâu thuẫn.
Tuy nhiên, đây không là điều kiện đủ.
Định lí 2.2.2. Nếu D = p là số nguyên tố thì (2.11) có nghiệm khi và chỉ
khi p = 2 hoặc p 6= 4k + 3.
Chứng minh. Nếu (2.11) có nghiệm thì từ Định lý 1 suy ra p = 2 hoặc p
6= 4k + 3. Đảo lại nếu p = 2 thì phương trình x2
= 2y2
= 1 có nghiệm (x;
y) = (1; 1). Xét p 1( mod 4). Xét phương trình Pell loại I
x2 Dy2 = 1 (2.12)
Gọi (a; b) là nghiệm nhỏ nhất của (2.12). Ta có a2 1 = pb2. Nếu a chẵn thì
b lẻ
do đó b
2
1( mod 4) ! a
2
1 + p 2( mod 4). Mâu thuẫn. Vậy a lẻ, b chẵn.
Đặt a = 2a1 + 1; b = 2b1. Ta có (a 1)(a + 1) = pb2
, a1(a1 + 1) = pb2
1.
Do p
nguyên tố và (a1; a1 +1) = 1 nên a1 = u
2
; a1 +1 = pv
2
hoặc a1 = pu
2
; a1
+1 = v
2
. Nếu a1 = u
2
; a1 + 1 = pv
2
! u
2
pv
2
= 1. Vậy (2.11) có nghiệm (u;
v). Nếu a1 = pu
2
; a1 + 1 = v
2
! v
2
pu
2
= 1. Vậy (v; u) là nghiệm của
(2.12). Suy ra u a ! a1 + 1 = v
2
v a = 2a1 + 1. Mâu thuẫn.
Định lí 2.2.3. Gọi (a; b) là nghiệm nhỏ nhất của (2.12). Khi đó (2.11) có nghiệm
khi và chỉ khi
8
> a = x2
+ Dy2
<
(2.13)
> b = 2xy
:
Viết thuê đề tài giá rẻ trọn gói - KB Zalo/Tele : 0973.287.149
Luanvanmaster.com – Cần Kham Thảo - Kết bạn Zalo/Tele : 0973.287.149
32
có nghiệm nguyên dương.
Hơn nữa nếu (2.13) hệ có nghiệm nó sẽ có nghiệm duy nhất. Nghiệm
duy nhất (x1; y1) này chính là nghiệm nhỏ nhất của (2.11).
Chứng minh. 1) Giả sử (2.11) có nghiệm. Gọi (x1; y1) là nghiệm nhỏ nhất
của (2.11). Đặt u = x1
2
+Dy2
1; v = 2x1; y1. Ta chứng minh u = a; v = b
do đó (x1; y1) chính là nghiệm của (2.13). Ta có u2
Dv2
= (x1
2
Dy2
1)2
= 1. Vậy (u; v) là nghiệm của (2.12). Suy ra u a; v b. Ta có
(a
2
Db
2
)(x1 Dy1
2
) = 1( 1) = 1
,(ax1 Dby1)
2
D(ay1 bx1)
2
= 1:
Dễ thấy (ax1 Dby1)2 > 0; (ay1 bx1)2 > 0. Do (x1; y1) là nghiệm của
(2.11) nên
(ax1 Dby1)
2
x1
2
, a
2
x1
2
+ D
2
b
2
y1
2
x1
2
2abDx1y1
! x1
2
(a2
1) + D2
b2
y1
2
2abDx1y1
! x1
2
Db
2
+ D
2
b
2
y
2
1 2abDx1y1
! b(x1
2
+ dy
2
1) 2ax1y1 ! bu av
! b
2
u
2
a
2
v
2
! b
2
(Dv
2
+ 1) v
2
(Db
2
+ 1)
! b v ! b = v ! u = a:
2) Đảo lại giả sử (u; v) là nghiệm của (2.13). Ta có a2
Db2
= 1 , (u2
+Dv2
)2
D(2uv)
2
= (u
2
Dv
2
)
2
= 1. Vậy u
2
Dv
2
= 1 hoặc u
2
Dv
2
= 1. Nếu
u
2
Dv
2
= 1 thì (u; v) là nghiệm của (2.12) do đó u a = u
2
+ Dv
2
. Mâu
thuẫn. Vậy u2
Dv2
= 1 do đó (2.11) có nghiệm (u; v).
Tiếp theo ta chứng minh (u; v) là nghiệm nhỏ nhất của (2.11). Giả sử (x1; y1) là
nghiệm nhỏ nhất của (2.11). Theo chứng minh ở 1) ta có a = u
2
+ Dv
2
= x1
2
+
p
Dy
2
1; b = 2uv = 2x1y1 ! u
2
+ Dv
2
+ 2uv = x1
2
+ Dy
2
1 + 2x1y1 ! (u +
v D)
2
= (x1 + y1)
2
! u = x1; v = y1. Định lý được chứng minh.
Viết thuê đề tài giá rẻ trọn gói - KB Zalo/Tele : 0973.287.149
Luanvanmaster.com – Cần Kham Thảo - Kết bạn Zalo/Tele : 0973.287.149
33
Ví dụ là một áp dụng của định lý 2. Nó cũng chỉ ra rằng điều kiện của
định lý 1 chỉ là điều kiện cần.
Ví dụ 2.2.4. Chứng minh rằng phương trình x
2
34y
2
= 1 vô nghiệm (Ở đây 34
= 2:17 không số chính phương và cũng không có ước nguyên tố dạng 4k + 3).
Giải Phương trình x
2
34y
2
= 1 có nghiệm nhỏ nhất là (a; b) = (35; 6). Xét
hệ
8
<35 = x
2
+ 34y
2
:6 = 2xy
Từ phương trình thứ nhất của hệ suy ra (x; y) = (1; 1). Tuy nhiên (1; 1) không thoả
mãn phương trình thứ hai. Vậy hệ vô nghiệm do đó theo định lý 2 phương trình
x
2
34y
2
= 1 vô nghiệm.
Ta thừa nhận định lý
Định lí 2.2.5. Phương trình Pell loại 2
x
2
Dy
2
= 1
p
có nghiệm khi và chỉ khi trong biểu diễn D thành liên
phân số p
D = [a; a1; a2; : : : ; an; 2a]
chu kỳ n là số lẻ. Trong trường hợp đó (pn 1; qn 1) là nghiệm nhỏ nhất
của phương trình ở đó.
pn 1
Cn 1 =
là giải phân thứ n 1.
p
Ví dụ 2.2.6. i) Xét phương trình x
2
13y
2
= 1. Ta có D = [3;1; 1; 1; 1; 6].
Chu kỳ n = 5 là số lẻ. Vậy phương trình có nghiệm. Ta tính giản phân
Viết thuê đề tài giá rẻ trọn gói - KB Zalo/Tele : 0973.287.149
Luanvanmaster.com – Cần Kham Thảo - Kết bạn Zalo/Tele : 0973.287.149
C4 = [3;1;1;1;1]
Viết thuê đề tài giá rẻ trọn gói - KB Zalo/Tele : 0973.287.149
Luanvanmaster.com – Cần Kham Thảo - Kết bạn Zalo/Tele : 0973.287.149
34
= 3 +
1
1
1 +
1
1 +
1
1 +
3 18
1
= 3 + =
5 5
Vậy nghiệm nhỏ nhất là (18; 5)
ii) Xét phương trình x
2
34y
2
= 1. Ta có
p
]. Chu kỳ n = 4
34 = [5;1;4;1;10
là số chẵn. Vậy phương trình vô nghiệm.
Định lí 2.2.7 (Công thức nghiệm). Giả sử phương trình Pell loại 2
x
2
Dy
2
= 1 (II)
có nghiệm. Gọi (a; b ) là nghiệm nhỏ nhất của nó. Khi đó dãy (xn; yn)¥
cho bởi
n=1
(a + b
p
)2n 1 + (a b
p
)2n 1
xn =
D D
(a + b
p 2
p
yn =
D)2n 1 + (a b D)2n 1
2pD
cho ta tất cả các nghiệm của (II).
Chứng minh. Giả sử (xn; yn) cho bởi công thức trên. Khi
đó xn + yn
p
D = (a + b
p
D)
2n 1
xn yn
p
= (a b
p
)2n 1
D D
! xn
2
Dyn
2
= (a2 Db
2
)
2n 1
= 1
Ngược lại giả sử (x; y) là một nghiệm của (2.11). Ta có
p p p
p
= s +t D
ởđó s = xa + Dyb ;t = ya + xb
! s2
Dt2
= (xa + Dyb )2
D(ya + xb )
2
Viết thuê đề tài giá rẻ trọn gói - KB Zalo/Tele : 0973.287.149
Luanvanmaster.com – Cần Kham Thảo - Kết bạn Zalo/Tele : 0973.287.149
35
= (x
2
Dy
2
)(a
2
Db
2
) = ( 1)( 1) = 1
Vậy (s;t) là nghiệm của phương trình Pell loại 1 s
2
Dt
2
= 1. Gọi (a; b) là
nghiệm nhỏ nhất của nó. Theo công thức nghiệm của phương trình Pell
loại 1 và định lý 3 tồn tại n 2 N sao cho
(x + y
p
D)(a + b
p
D) = s +t
p
D = (a + b
p
D)
n
= (a
2
+ Db
2
+ 2ab
p
D)
n
= (a + b
p
D)
2
)
n
= (a + b
p
D)
2n
! x + y
p
D = (a + b
p
D)2n 1
= xn + yn
p
D
! x = xn; y = yn
Một số bài toán chọn lọc
Bài toán 2.2.8. Số nguyên dương S được gọi là số Heron nếu nó là diện tích
của một tam giác có ba cạnh là ba số nguyên liên tiếp. Chứng minh rằng S là
số Heron khi và chỉ khi S khi là số hạng của dãy (Sn); n 1 xác định bởi
S0 = 0; S1 = 6; S2 = 84; Sn+2 = 14Sn+1 Sn
Lời giải. Gọi S là diện tích của tam giác có ba cạnh là x 1; x; x + 1 với x > 2; x 2
N . Theo công thức Heron.
1
x
q
3(x
2
4) ! 16S
2
= 3x
2
(x
2
(2.14)
S = 1)
4
Vậy S là số Heron khi và chỉ khi phương trình (2.14) có nghiệm nguyên
dương(S; x). Dễ thấy x phải chẵn. Đặt x = 2y ta có
16S
2
= 3x
2
(x
2
1) , S
2
= 3y
2
(y
2
1)
! S = y
q
3(y
2
1) ! 3(y
2
= 1) = h
2
Viết thuê đề tài giá rẻ trọn gói - KB Zalo/Tele : 0973.287.149
Luanvanmaster.com – Cần Kham Thảo - Kết bạn Zalo/Tele : 0973.287.149
36
! h = 3z ! 3(y
2
1) = 9z
2
! y
2
3z
2
= 1; S = 3yz
Ngược lại nếu (y; z) là nghiệm của phương trình Pell
y2
3z2
= 1 (2.15)
thì x = 2y; y > 1; S = 3yz là nghiệm của (2.14) Nghiệm nhỏ nhất của
(2.15) là (2; 1) Vậy tất cả nghiệm của (2.15) (yn; zn) cho bởi
(2 +
p
)
n
+ (2 p )n
yn =
3 3
(2 +
p
)
n
2
p )
n
zn =
3
p
(2 3
2 3
Do đó
p
p )n (7 4
p
Sn = 3ynzn = 3 ((7 + )
n
)
3 3
4
! Sn+2 = 14Sn+1 Sn; S0 = 0; S1 = 6; S2 = 84
Bài toán 2.2.9. Tìm tất cả các số nguyên dương n sao cho 2n + 1 và 3n
+ 1 đều là số chính phương.
Lời giải. Vì (2n + 1; 3n + 1) = 1 nên 2n + 1 và 3n + 1 đều là số chính
phương khi và chỉ khi (2n + 1)(3n + 1) = y2
; y 2 N . Suy ra (12n + 5)2
24y2
=
1. Đặt x = 12n + 5 ta có x2
24y2
= 1. Gọi (xk; yk) là nghiệm của nó. Nghiệm
nhỏ nhất của phương trình Pell này là (5; 1). Do đó (xk) cho bởi hệ thức.
x0 = 1; x1 = 5; xk+2 = 10xk+1 xk:
Dễ chứng minh xk 5 (mod 12) khi vào chỉ khi k lẻ. Vậy n = nk; k 1 ở đó
nk =
x
2k+1
5
12
Ta xác định hệ thức truy hồi của dãy (nk)
Viết thuê đề tài giá rẻ trọn gói - KB Zalo/Tele : 0973.287.149
Luanvanmaster.com – Cần Kham Thảo - Kết bạn Zalo/Tele : 0973.287.149
37
Đặt uk = x2k+1 = 12nk + 5. Ta có
x
2k+3
= 10x
2k+2
x
2k+1
= 10(10x
2k+1
x
2k
) x
2k+1
=
100x
2k+1
10x
2k
x
2k+1
= 99x
2k+1
= x
2k+1
x
2k 1
= 98x
2k+1
x
2k 1
! uk+2 = 98uk+1 uk
, 12nk+2 + 5 = 98(12nk+1 + 5) = 12nk 5
nk+2 = 98nk+1 nk + 40
với n1 = 40; n2 = 3960
Bài toán 2.2.10. Cho dãy (xn; yn) xác định như sau (x0; y0) = (0; 1); (x1;
y1) = (3; 5) và
8
>
xn+1 = 3xn + 2yn + 1
<
(2.16)
> y = 4x + 3y + 2
:
n+1 n n
Chứng minh rằng (xn; yn) là tất cả các nghiệm nguyên dương của phương trình
x2
+ (x + 1)2 = y2
Lời giải. Phương trình đã cho tương đương với
(2x + 1)2
2y2
= 1 (2.17)
Đặt u = 2x +1; ! u
2
2y
2
= 1. Nghiệm nhỏ nhất của phương trình này là
(1:1). Do vậy dãy nghiệm (un; yn) cho bởi
(1 +
p
)
2n+1
+ (1 p )2n+1
un =
2 2
(1 +
p
)2n+1
2
p )2n+1
yn =
2
p
(1 2
2 2
Từ đó
u0 = 1; u1 = 7; uk+2 = 6uk+1 uk
Viết thuê đề tài giá rẻ trọn gói - KB Zalo/Tele : 0973.287.149
Luanvanmaster.com – Cần Kham Thảo - Kết bạn Zalo/Tele : 0973.287.149
38
y0 = 1; y1 = 5; yk+2 = 6yk+1 yk
Ta có un = 2xk+2 +1 ! 2xk+2 +1 = 6(2nk+1 +1) 2nk 1 ! x0 = 0; x1 = 3; xk+2
= 6xk+1 xk + 2. Thành thử dãy nghiệm (xn; yn) của (2.17) cho bởi
x0 = 0; x1 = 3; xn+2 = 6xn+1 xn + 2
y0 = 1; y1 = 5; yn+2 = 6yn+1 = yk
Thành thử ta chỉ cần chứng minh dãy (2.16) thoả mãn quan hệ
x
n+2
= 6x
n+1
x
n
+ 2
y
n+2
= 6y
n+1
y
k
Thật vậy đặt zn = 2xn + 1. Khi đó dễ kiểm tra
zn+1 = 3zn + 4yn
yn+1 = 2zn + 3yn
! zn+2 = 3zn + 1 + 4yn+1 = 3zn+1 + 8zn + 12yn
= 3zn+1 + 8zn + 3zn+1 9zn = 6zn+1 zn
! 2xn+2 + 1 = 6(2xn+1 + 1) 2xn 1
! xn+2 = 6xn+1 xn + 2
Tương tự yn+2 = 6yn+1 yk
2.3 Phương trình Pythagoras
Trong mục này chúng ta sẽ đi tìm tất cả các nghiệm nguyên dương của
phương trình
x
2
+ y
2
= z
2
(2.18)
và xét một số ứng dụng của nó. Những phương trình Diophantine này
được gọi là các phương trình Pythagoras.
Viết thuê đề tài giá rẻ trọn gói - KB Zalo/Tele : 0973.287.149
Luanvanmaster.com – Cần Kham Thảo - Kết bạn Zalo/Tele : 0973.287.149
39
Định nghĩa 2.3.1. Một bộ ba số nguyên dương (x; y; z) thoả mãn
phương trình (2.18) gọi là một bộ ba số Pythagoras.
Các bộ ba số Pythagora biểu thị độ dài các cạnh của một tam giác vuông.
Từ một bộ ba số Pythagoras (x; y; z) nào đó ta suy ra dễ dàng một tập hợp vô
hạn các bộ ba số Pythagoras khác (tx;ty;tz) với t là số nguyên dương.
Ngược lại, cho (x; y; z) là một bộ ba số Pythagoras bất kỳ và giả sử d
= (x; y; z). Khi đó (x1; y1; z1) cũng là một bộ ba số Pythagoras, ở đó
x
1 =x ;
y
1 = y ;
z
1 =z ; và (x1; y1; z1) = 1:
d d d
Định nghĩa 2.3.2. Một bộ ba số Pythagoras (x; y; z) là nguyên thuỷ nếu (x; y; z) = 1:
Rõ ràng từ lý luận trên ta chỉ cần đi tìm các bộ ba số Pythagoras nguyên thuỷ.
Dễ dàng nhận thấy nếu (x; y; z) là một bộ ba số Pythagoras nguyên thuỷ thì
chúng đôi một nguyên tố cùng nhau. Thật vậy, giả sử d = (x; y). Khi đó d
2
là
ước của z
2
nên d là ước của z do đó d là ước chung của x, y, z. Vậy d = 1.
Định lí sau đây cho công thức mô tả tất cả các bộ ba số Pythagoras
nguyên thuỷ.
Định lí 2.3.3. Giả sử (x; y; z) là một bộ ba số Pythagoras nguyên thuỷ. Khi đó x
và y có tính chẵn lẻ khác nhau. Nếu x chẵn, y lẻ chẳng hạn (x; y; z) có dạng
x = 2mn; y = m
2
n
2
; n = m
2
+ n
2
trong đó m, n là hai số nguyên đương nguyên tố cùng nhau, có tính chẵn
lẻ khác nhau.
Đảo lại, nếu m, n là hai số nguyên dương nguyên tố cùng nhau, một chẵn, một
lẻ thì ba bộ (x; y; z) xác định như trên là một bộ ba số Pythagoras nguyên thuỷ.
Chứng minh. Trước hết vì (x; y) = 1 nên x và y không cùng chẵn. Hai số x và y
cũng không thể cùng lẻ vì nếu như vậy thì z2
= x2
+ y2
2 (mod 4), điều này
không thể có vì một số chính phương chỉ đồng dư 0 hoặc 1 theo modulo 4.
Viết thuê đề tài giá rẻ trọn gói - KB Zalo/Tele : 0973.287.149
Luanvanmaster.com – Cần Kham Thảo - Kết bạn Zalo/Tele : 0973.287.149
40
Giả sử x chẵn, y lẻ. Khi đó z lẻ và x
2
= z
2
y
2
(z + y)(z y), điều này tương
đương với 2
2
= 2 2 : (2.19)
x z + y z y
z+y z y z+y
= m
2
và
Dễ thấy rằng ; = 1 vì (z; y) = 1. Suy ra tồn tại m; n 2 N để
2 2 2
z y
= n
2
. Từ đó x = 2mn, y = m
2
n
2
, z = m
2
+ n
2
. Mặt khác (m
2
; n
2
) = 1 nên
2
(m; n) = 1. Hơn nữa vì y và z lẻ nên m, n có tính chẵn lẻ khác nhau.
Bây giờ ta chứng minh phần đảo của định trong định lí trên. Dễ dàng kiểm tra
rằng x, y, z xác định theo công thức trên là bộ ba số Pythagoras.
Ta còn phải chứng minh đó là bộ ba số Pythagoras nguyên thuỷ. Giả sử d =
(y; z): Khi đó vì y, z lẻ nên d lẻ. Ta có d j y + z = 2m2
suy ra
d j m2
và d j z y = 2n2
suy ra d j n2
:
Vì (m; n) = 1 nên từ đó d = 1. Vậy (y; z) = 1 suy ra (x; y; z) = 1. Định lí
được chứng minh.
Sau đây là áp dụng quan trọng của Định lí 2.3.3.
Định lí 2.3.4. Không tồn tại hai số tự nhiên x và y để tổng các bình phương
và hiệu các bình phương của chúng đều là các số chính phương.
Chứng minh. Ta chứng minh bằng phản chứng. Giả sử tồn tại các số
nguyên dương x, y, có tính chất đã nêu.
Trong các cặp số (x; y) ta chọn (x0; y0) là cặp số mà x nhỏ nhất. Giả sử
8
x0
2
+ y0
2
= z0
2
;
2 2 2 (2.20)
<
x0 y0 = t0 ;
:
với z0; t0 N .
Ta có (x0; y0) = 1. Thật vậy, giả sử d = (x0; y0). Từ (2.20) suy
ra d2
d2
j t0
2
. Từ đó d j z0 , d j t0 . Đặt
x0
y1
y0
z1 = y
z0
t1
t0
; = ; ; = :
d d d d
j z
2
0 và
Viết thuê đề tài giá rẻ trọn gói - KB Zalo/Tele : 0973.287.149
Luanvanmaster.com – Cần Kham Thảo - Kết bạn Zalo/Tele : 0973.287.149
41
Ta có
x1
2
+ y
2
1 = z
2
1; x1
2
y
2
1 = t1
2
:
Suy ra (x1; y1) là một cặp thoả mãn tính chất đã nêu. Do đó
x1
2
+ y2
1 x0
2
+ y2
0 = d(x1
2
+ y2
1):
Vậy d = 1:
Từ (2.20) suy ra z2
0 +t0
2
= 2x0
2
. Do đó z0 và t0 có cùng tính chẵn lẻ.
Đặt
z0 +t0 z0 t0
u = ; v =
2 2
với u, v là các số nguyên dương.
Khi đó
u + v = z0; u v = t0vu2
+ v2
= x0
2
: (2.21)
Ta có (u; v) = 1, vì nếu d = (u; v) thì d
2
j x0
2
. Mặt khác d j u + v = z0, suy
ra d
2
j z
2
0 x0
2
= y
2
0 suy ra d j y0. Vậy d = 1:
Theo (2.21) thì (u; v; x0) là một bộ ba số Pythagoras nguyên thuỷ.
Theo Định lí 2.3.3 tồn tại các số nguyên dương m, n chẵn lẻ khác nhau
với (m; n) = 1, m > n sao cho
hoặc
u = 2mn;
u = 2m2
v = m2 n2;
n2; v = 2mn:
Trong mọi trường hợp, uv = 2mn(m2 n2). Ta lại có
2y = (u + v)2 (u v)2 = 4uv = 8mn(m2 n2)
suy ra y = 4mn(m2 n2), tức là y0 = 2k. Vậy
mn(m2 n2) = k2: (2.22)
Ta có (m; n) = 1 nên (m + n; m) = 1 và (m n; m) = 1. Vậy
(m2
n2; m) = 1:
Viết thuê đề tài giá rẻ trọn gói - KB Zalo/Tele : 0973.287.149
Luanvanmaster.com – Cần Kham Thảo - Kết bạn Zalo/Tele : 0973.287.149
42
Tương tự, (m2
n2
; n) = 1. Do điều này nên từ (2.3) suy ra tồn tại a; b; c 2 N
để
m = a2; n = b2;
m
2 n2 = c2: (2.23)
Gọi d = (m + n; m n). Do m, n chẵn lẻ khác nhau nên m + n, m n lẻ, từ đó d lẻ.
Vì d j m + n, d j m n nên d j 2m, d j 2n suy ra d j m, d j n (do d lẻ). Suy ra d = 1.
Do điều này nên từ (2.23) suy ra tồn tại r; s 2 N để
m n = r
2
; m + n = s
2
:
Vậy
a2 + b2 = s
2
;
a
2 b2 = r
2
;
tức là cặp (a; b) thoả mãn tính chất đã nêu trong định lí.
Mặt khác
z0 +t0
a
2
+ b
2
= m + n 2m 2mn u = < z0 z0
2
= x0
2
= y0
2
:
2
Điều này trái với cách chọn cặp (x0; y0). Định lí được chứng minh xong.
Một số bài toán chọn lọc
Bài toán 2.3.5 (Đề dự tuyển thi Toán quốc tế năm 1979). Chứng minh
rằng không tồn tại hình chóp tứ giác đều mà các cạnh, diện tích toàn
phần và thể tích của nó đều là các số nguyên.
Chứng minh. Giả sử g, f , h, S và V theo thứ tự là cạnh đáy, cạnh bên,
chiều cao, diện tích toàn phần và thể tích của một hình chóp tứ giác đều
và chúng là những số nguyên. Ta có
f = r V = g
3
h; S = g2 + 2g r
h2 + 2 ; h2 + 4 :
g
2
2 g
2
Vì g, S là số nguyên nên 2g
q
h
2
+
g
2
là số nguyên. Kí hiệu
4
x = 2g2
r
h2 + 4 ; y = g3:
g
2
Viết thuê đề tài giá rẻ trọn gói - KB Zalo/Tele : 0973.287.149
Luanvanmaster.com – Cần Kham Thảo - Kết bạn Zalo/Tele : 0973.287.149
43
Dễ thấy x; y 2 N . Ta có
x
2
y2 = 4g4 22 +
g
2
g
6
= 4h
2
g
4
= (2g
2
h)
2
4
là số chính phương, vì V =
g
3
2h
2 N . Do đó g
2
2 N . Mặt khác,
g2
x2 + y2 = 4g4h2 + 2g6 = 4g4 h2 + = 4g4 f 2 = (2g2 f )2:
Vì 2g
2
f 2 N nên x
2
+ y
2
là số chính phương. Song điều này mâu thuẫn
với Định lí 2.3.4. Ta có điều cần chứng minh.
Bài toán 2.3.6. Chứng minh rằng phương trình
x
4
y
4
= z
2
không có nghiệm nguyên dương.
Chứng minh. Ta sẽ chứng minh bài toán này bằng phương pháp phản chứng. Giả
sử (x; y; z) là nghiệm nguyên dương của phương trình đã cho và d = (x; y). Ta có
x = dx0; y = dy0; (x0; y0) = 1:
Thay vào phương trình cho ta
d
4
(x0
4
y0
4
) = z
2
suy ra d
2
j z:
Đặt z = d
2
z0 suy ra x0
4
y0
4
= z0
2
, tức là (x0
2
y0
2
)(x0
2
+ y0
2
) = z0
2
: Nếu x0 và y0 chẵn
lẻ khác nhau thì (x0
2
y0
2
; x0
2
+ y0
2
) = 1 bởi vì nếu d1 j (x0
2
y0
2
) và d1 j (x0
2
+ y0
2
) thì
d1 j 2y0
2
và d1 j 2x0
2
. Vậy d1 = 1 (do d1 lẻ).
Do đó x0
2
y
2
0 = a
2
; x0
2
y
2
0 = b
2
, trái với Định lí 2.3.4.
Nếu x và y cùng lẻ thì x2
+y2
= 2a; x2
+y2
= 2b Suy ra x2
= a+b, y2
= ab.
0 0 0 0 0 0 0 0
Ta có 4ab = z
2
suy ra ab = z
2
. Vì (x ; y ) = 1 nên (a; b) = 1. Do đó a = a
2
, b = b
2
.
0 1 0 0 1 1
Viết thuê đề tài giá rẻ trọn gói - KB Zalo/Tele : 0973.287.149
Luanvanmaster.com – Cần Kham Thảo - Kết bạn Zalo/Tele : 0973.287.149
Vậy a
2
+ b
2
= x
2
, a
2
b2
= y2
, và điều này mâu thuẫn với Định lí
2.3.4. 110110
Viết thuê đề tài giá rẻ trọn gói - KB Zalo/Tele : 0973.287.149
Luanvanmaster.com – Cần Kham Thảo - Kết bạn Zalo/Tele : 0973.287.149
44
Bài toán 2.3.7. Chứng minh rằng phương trình
x
4
+ y
4
= z
2
không có nghiệm nguyên dương.
Chứng minh. Giả sử trái lại, phương trình đã cho có nghiệm nguyên dương.
Gọi (x0; y0) là nghiệm nguyên dương sao cho tổng x là nhỏ nhất. Lập luận
tương tự như trong chứng minh Định lí 2.3.4 ta thấy (x0; y0) = 1. Như vậy (x0
2
;
y
2
0; z0) là một bộ ba số Pythagoras nguyên thuỷ. Không làm giảm tổng quát ta
giả sử x0 lẻ. Theo Định lí 2.3.3 tồn tại m, n với (m; n) = 1 sao cho
n
x = m
2
n
2
; y = 2mn;z0 = m
2
+ n
2
:
(2.24)
Vì (m; n) = 1 nên từ (2.24) ta có (x0; n; m) là một bộ ba số Pythagoras
nguyên thuỷ. Vậy lại theo Định lí 2.3.3 tồn tại a, b với (a; b) = 1 sao cho
n
x0 = a
2
b
2
; n = 2ab;m = a
2
+ b
2
:
(2.25)
Ta có y
2
0s = 2mn = 4abm suy ra abm = y
2
1 (ở đây y0 = 2y1) suy ra a =
a
2
1, b = b
2
1, m = m
2
1. Thay vào (2.25) ta có m
2
1 = a
4
1 + b
4
1. Mặt khác
a
4
1 + b
4
1 = m
2
1 = m < m
2
+ n
2
= z0z
2
0 = x0
4
+ y
4
0:
Điều này trái với cách chọn (x0; y0). Vậy bài toán được chứng minh.
Nhận xét 2.3.8. Qua chứng minh 2.3.4. và lời giải của các Bài toán 2.3.5, 2.3.6,
2.3.7, ta có thể nhận thấy một phương pháp chung sau đây trong lí thuyết phương
trình Diophantine: Để chứng minh một phương trình Diophantine đã cho không có
nghiệm nguyên dương, ta hãy giả sử nó có và khi đó sẽ có một nghiệm nhỏ nhất
(theo một nghĩa nào đó). Sau đó ta sẽ cố gắng kiến thiết một nghiệm nhỏ hơn
nghiệm nhỏ nhất và như vậy ta dẫn đến mâu thuẫn.
Viết thuê đề tài giá rẻ trọn gói - KB Zalo/Tele : 0973.287.149
Luanvanmaster.com – Cần Kham Thảo - Kết bạn Zalo/Tele : 0973.287.149
45
Chương 3
Liên phân số và ứng dụng
trong phương trình Diophantine

More Related Content

Similar to Luận văn thạc sĩ toán học - Một số lớp phương trình diophantine.doc

Luận văn: Nghiên cứu DIDACTIC về dạy học các bài toán tối ưu trong chủ đề giả...
Luận văn: Nghiên cứu DIDACTIC về dạy học các bài toán tối ưu trong chủ đề giả...Luận văn: Nghiên cứu DIDACTIC về dạy học các bài toán tối ưu trong chủ đề giả...
Luận văn: Nghiên cứu DIDACTIC về dạy học các bài toán tối ưu trong chủ đề giả...Viết thuê trọn gói ZALO 0934573149
 
Kỹ thuật lập trình (khoa toán tin học)
Kỹ thuật lập trình (khoa toán tin học)Kỹ thuật lập trình (khoa toán tin học)
Kỹ thuật lập trình (khoa toán tin học)nhok_lovely
 
Sáng kiến kinh nghiệm đổi mới phương pháp giảng dạy Toán học
Sáng kiến kinh nghiệm đổi mới phương pháp giảng dạy Toán họcSáng kiến kinh nghiệm đổi mới phương pháp giảng dạy Toán học
Sáng kiến kinh nghiệm đổi mới phương pháp giảng dạy Toán họcHọc Tập Long An
 
Decuongontaptoan8hk21657 150928065453-lva1-app6892
Decuongontaptoan8hk21657 150928065453-lva1-app6892Decuongontaptoan8hk21657 150928065453-lva1-app6892
Decuongontaptoan8hk21657 150928065453-lva1-app6892Phượng Hoàng
 
De cuong on_tap_toan_8_hk2_1657
De cuong on_tap_toan_8_hk2_1657De cuong on_tap_toan_8_hk2_1657
De cuong on_tap_toan_8_hk2_1657Phượng Hoàng
 
Giải tích các hàm nhiều biến.pdf
Giải tích các hàm nhiều biến.pdfGiải tích các hàm nhiều biến.pdf
Giải tích các hàm nhiều biến.pdfMan_Ebook
 
Luận án tiến sĩ toán học những khía cạnh số học của lí thuyết phân bố giá trị
Luận án tiến sĩ toán học những khía cạnh số học của lí thuyết phân bố giá trịLuận án tiến sĩ toán học những khía cạnh số học của lí thuyết phân bố giá trị
Luận án tiến sĩ toán học những khía cạnh số học của lí thuyết phân bố giá trịhttps://www.facebook.com/garmentspace
 
Luận án tiến sĩ toán học ngưỡng chính tắc của hàm chỉnh hình và hàm đa điều h...
Luận án tiến sĩ toán học ngưỡng chính tắc của hàm chỉnh hình và hàm đa điều h...Luận án tiến sĩ toán học ngưỡng chính tắc của hàm chỉnh hình và hàm đa điều h...
Luận án tiến sĩ toán học ngưỡng chính tắc của hàm chỉnh hình và hàm đa điều h...https://www.facebook.com/garmentspace
 
Kỹ thuật lập trình - PGS.TS. Phạm Thế Bảo
Kỹ thuật lập trình - PGS.TS. Phạm Thế BảoKỹ thuật lập trình - PGS.TS. Phạm Thế Bảo
Kỹ thuật lập trình - PGS.TS. Phạm Thế BảoNguyen Van Nghiem
 

Similar to Luận văn thạc sĩ toán học - Một số lớp phương trình diophantine.doc (20)

Một số lớp đa thức hoán vị trên trường hữu hạn đặc số chẵn.doc
Một số lớp đa thức hoán vị trên trường hữu hạn đặc số chẵn.docMột số lớp đa thức hoán vị trên trường hữu hạn đặc số chẵn.doc
Một số lớp đa thức hoán vị trên trường hữu hạn đặc số chẵn.doc
 
Luận văn: Nghiên cứu DIDACTIC về dạy học các bài toán tối ưu trong chủ đề giả...
Luận văn: Nghiên cứu DIDACTIC về dạy học các bài toán tối ưu trong chủ đề giả...Luận văn: Nghiên cứu DIDACTIC về dạy học các bài toán tối ưu trong chủ đề giả...
Luận văn: Nghiên cứu DIDACTIC về dạy học các bài toán tối ưu trong chủ đề giả...
 
Luận văn thạc sĩ - Bất đẳng thức với hàm lồi bộ phận và ứng dụng.doc
Luận văn thạc sĩ - Bất đẳng thức với hàm lồi bộ phận và ứng dụng.docLuận văn thạc sĩ - Bất đẳng thức với hàm lồi bộ phận và ứng dụng.doc
Luận văn thạc sĩ - Bất đẳng thức với hàm lồi bộ phận và ứng dụng.doc
 
Xap xỉ diophantine và phân so liên tục trong giải Phương trình pell.docx
Xap xỉ diophantine và phân so liên tục  trong giải Phương trình pell.docxXap xỉ diophantine và phân so liên tục  trong giải Phương trình pell.docx
Xap xỉ diophantine và phân so liên tục trong giải Phương trình pell.docx
 
Về phương trình hàm Loại giá trị trung bình và áp dụng.docx
Về phương trình hàm Loại giá trị trung bình và áp dụng.docxVề phương trình hàm Loại giá trị trung bình và áp dụng.docx
Về phương trình hàm Loại giá trị trung bình và áp dụng.docx
 
Luận văn thạc sĩ - Xấp xỉ hàm đa điều hòa dưới bởi hàm Green đa cực.doc
Luận văn thạc sĩ - Xấp xỉ hàm đa điều hòa dưới bởi hàm Green đa cực.docLuận văn thạc sĩ - Xấp xỉ hàm đa điều hòa dưới bởi hàm Green đa cực.doc
Luận văn thạc sĩ - Xấp xỉ hàm đa điều hòa dưới bởi hàm Green đa cực.doc
 
Xấp xỉ hàm đa điều hòa dưới Bởi hàm green đa cực.doc
Xấp xỉ hàm đa điều hòa dưới Bởi hàm green đa cực.docXấp xỉ hàm đa điều hòa dưới Bởi hàm green đa cực.doc
Xấp xỉ hàm đa điều hòa dưới Bởi hàm green đa cực.doc
 
Đa Thức Trong Các Bài Toán Thi Học Sinh Giỏi.docx
Đa Thức Trong Các Bài Toán Thi Học Sinh Giỏi.docxĐa Thức Trong Các Bài Toán Thi Học Sinh Giỏi.docx
Đa Thức Trong Các Bài Toán Thi Học Sinh Giỏi.docx
 
Luận Văn Lý Thuyết Nevanlinna Và Ứng Dụng.
Luận Văn Lý Thuyết Nevanlinna Và Ứng Dụng.Luận Văn Lý Thuyết Nevanlinna Và Ứng Dụng.
Luận Văn Lý Thuyết Nevanlinna Và Ứng Dụng.
 
Kỹ thuật lập trình (khoa toán tin học)
Kỹ thuật lập trình (khoa toán tin học)Kỹ thuật lập trình (khoa toán tin học)
Kỹ thuật lập trình (khoa toán tin học)
 
Sáng kiến kinh nghiệm đổi mới phương pháp giảng dạy Toán học
Sáng kiến kinh nghiệm đổi mới phương pháp giảng dạy Toán họcSáng kiến kinh nghiệm đổi mới phương pháp giảng dạy Toán học
Sáng kiến kinh nghiệm đổi mới phương pháp giảng dạy Toán học
 
Phương Pháp Bình Phương Nhỏ Nhất Và Ứng Dụng.doc
Phương Pháp Bình Phương Nhỏ Nhất Và Ứng Dụng.docPhương Pháp Bình Phương Nhỏ Nhất Và Ứng Dụng.doc
Phương Pháp Bình Phương Nhỏ Nhất Và Ứng Dụng.doc
 
Decuongontaptoan8hk21657 150928065453-lva1-app6892
Decuongontaptoan8hk21657 150928065453-lva1-app6892Decuongontaptoan8hk21657 150928065453-lva1-app6892
Decuongontaptoan8hk21657 150928065453-lva1-app6892
 
De cuong on_tap_toan_8_hk2_1657
De cuong on_tap_toan_8_hk2_1657De cuong on_tap_toan_8_hk2_1657
De cuong on_tap_toan_8_hk2_1657
 
Giải tích các hàm nhiều biến.pdf
Giải tích các hàm nhiều biến.pdfGiải tích các hàm nhiều biến.pdf
Giải tích các hàm nhiều biến.pdf
 
Luận án tiến sĩ toán học những khía cạnh số học của lí thuyết phân bố giá trị
Luận án tiến sĩ toán học những khía cạnh số học của lí thuyết phân bố giá trịLuận án tiến sĩ toán học những khía cạnh số học của lí thuyết phân bố giá trị
Luận án tiến sĩ toán học những khía cạnh số học của lí thuyết phân bố giá trị
 
Số Phức Và Ứng Dụng Vào Giải Toán Phổ Thông Trung Học.doc
Số Phức Và Ứng Dụng Vào Giải Toán Phổ Thông Trung Học.docSố Phức Và Ứng Dụng Vào Giải Toán Phổ Thông Trung Học.doc
Số Phức Và Ứng Dụng Vào Giải Toán Phổ Thông Trung Học.doc
 
Luận án tiến sĩ toán học ngưỡng chính tắc của hàm chỉnh hình và hàm đa điều h...
Luận án tiến sĩ toán học ngưỡng chính tắc của hàm chỉnh hình và hàm đa điều h...Luận án tiến sĩ toán học ngưỡng chính tắc của hàm chỉnh hình và hàm đa điều h...
Luận án tiến sĩ toán học ngưỡng chính tắc của hàm chỉnh hình và hàm đa điều h...
 
Ứng Dụng Hình Học Tính Toán Để Xác Định Một Miền Chứa Điểm Cho Trƣớc.doc
Ứng Dụng Hình Học Tính Toán Để Xác Định Một Miền Chứa Điểm Cho Trƣớc.docỨng Dụng Hình Học Tính Toán Để Xác Định Một Miền Chứa Điểm Cho Trƣớc.doc
Ứng Dụng Hình Học Tính Toán Để Xác Định Một Miền Chứa Điểm Cho Trƣớc.doc
 
Kỹ thuật lập trình - PGS.TS. Phạm Thế Bảo
Kỹ thuật lập trình - PGS.TS. Phạm Thế BảoKỹ thuật lập trình - PGS.TS. Phạm Thế Bảo
Kỹ thuật lập trình - PGS.TS. Phạm Thế Bảo
 

More from Dịch vụ viết thuê đề tài trọn gói ☎☎☎ Liên hệ ZALO/TELE: 0973.287.149 👍👍

More from Dịch vụ viết thuê đề tài trọn gói ☎☎☎ Liên hệ ZALO/TELE: 0973.287.149 👍👍 (20)

Phân tích các yếu tố ảnh hưởng đến lòng trung thành của nhân viên tại khách s...
Phân tích các yếu tố ảnh hưởng đến lòng trung thành của nhân viên tại khách s...Phân tích các yếu tố ảnh hưởng đến lòng trung thành của nhân viên tại khách s...
Phân tích các yếu tố ảnh hưởng đến lòng trung thành của nhân viên tại khách s...
 
Nghiên cứu về phát triển hệ thống kênh phân phối sản phẩm của các doanh nghiệ...
Nghiên cứu về phát triển hệ thống kênh phân phối sản phẩm của các doanh nghiệ...Nghiên cứu về phát triển hệ thống kênh phân phối sản phẩm của các doanh nghiệ...
Nghiên cứu về phát triển hệ thống kênh phân phối sản phẩm của các doanh nghiệ...
 
CƠ SỞ LÝ LUẬN VỀ THƯƠNG HIỆU.docx
CƠ SỞ LÝ LUẬN VỀ THƯƠNG HIỆU.docxCƠ SỞ LÝ LUẬN VỀ THƯƠNG HIỆU.docx
CƠ SỞ LÝ LUẬN VỀ THƯƠNG HIỆU.docx
 
Cơ sở lý luận của việc nâng cao chất lượng phục vụ tại bộ phận đón tiếp của k...
Cơ sở lý luận của việc nâng cao chất lượng phục vụ tại bộ phận đón tiếp của k...Cơ sở lý luận của việc nâng cao chất lượng phục vụ tại bộ phận đón tiếp của k...
Cơ sở lý luận của việc nâng cao chất lượng phục vụ tại bộ phận đón tiếp của k...
 
Cơ sở lý luận về phát triển thị trƣờng khách inbound dưới góc độ marketing củ...
Cơ sở lý luận về phát triển thị trƣờng khách inbound dưới góc độ marketing củ...Cơ sở lý luận về phát triển thị trƣờng khách inbound dưới góc độ marketing củ...
Cơ sở lý luận về phát triển thị trƣờng khách inbound dưới góc độ marketing củ...
 
Cơ sở lý luận về thị trường và sử dụng marketing nhằm mở rộng thị trường của ...
Cơ sở lý luận về thị trường và sử dụng marketing nhằm mở rộng thị trường của ...Cơ sở lý luận về thị trường và sử dụng marketing nhằm mở rộng thị trường của ...
Cơ sở lý luận về thị trường và sử dụng marketing nhằm mở rộng thị trường của ...
 
Tính toán thiết kế chế tạo và vận hành thử nghiệm hệ thống cấp đông I-Q-F thẳ...
Tính toán thiết kế chế tạo và vận hành thử nghiệm hệ thống cấp đông I-Q-F thẳ...Tính toán thiết kế chế tạo và vận hành thử nghiệm hệ thống cấp đông I-Q-F thẳ...
Tính toán thiết kế chế tạo và vận hành thử nghiệm hệ thống cấp đông I-Q-F thẳ...
 
Tính toán, thiết kế máy sấy bơm nhiệt sấy thanh long cắt lát với năng suất 20...
Tính toán, thiết kế máy sấy bơm nhiệt sấy thanh long cắt lát với năng suất 20...Tính toán, thiết kế máy sấy bơm nhiệt sấy thanh long cắt lát với năng suất 20...
Tính toán, thiết kế máy sấy bơm nhiệt sấy thanh long cắt lát với năng suất 20...
 
Nghiên cứu nhiệt phân gỗ nhằm nâng cao chất lượng sản phẩm than hoa.doc
Nghiên cứu nhiệt phân gỗ nhằm nâng cao chất lượng sản phẩm than hoa.docNghiên cứu nhiệt phân gỗ nhằm nâng cao chất lượng sản phẩm than hoa.doc
Nghiên cứu nhiệt phân gỗ nhằm nâng cao chất lượng sản phẩm than hoa.doc
 
Hoàn thiện quy trình sản xuất thanh long sấy bằng phương pháp sấy đối ...
Hoàn thiện quy trình sản xuất thanh long sấy bằng phương pháp sấy đối ...Hoàn thiện quy trình sản xuất thanh long sấy bằng phương pháp sấy đối ...
Hoàn thiện quy trình sản xuất thanh long sấy bằng phương pháp sấy đối ...
 
Nghiên cứu ứng dụng hệ điều khiển dự báo để điều khiển mức nước bao hơi của n...
Nghiên cứu ứng dụng hệ điều khiển dự báo để điều khiển mức nước bao hơi của n...Nghiên cứu ứng dụng hệ điều khiển dự báo để điều khiển mức nước bao hơi của n...
Nghiên cứu ứng dụng hệ điều khiển dự báo để điều khiển mức nước bao hơi của n...
 
ĐỒ ÁN - BÁO CÁO MÔ HÌNH KHO LẠNH DÀN TRẢI.doc
ĐỒ ÁN - BÁO CÁO MÔ HÌNH KHO LẠNH DÀN TRẢI.docĐỒ ÁN - BÁO CÁO MÔ HÌNH KHO LẠNH DÀN TRẢI.doc
ĐỒ ÁN - BÁO CÁO MÔ HÌNH KHO LẠNH DÀN TRẢI.doc
 
ĐỒ ÁN - Tính toán thiết kế máy sấy khoai lang năng suất 100 kg mẻ.doc
ĐỒ ÁN - Tính toán thiết kế máy sấy khoai lang năng suất 100 kg mẻ.docĐỒ ÁN - Tính toán thiết kế máy sấy khoai lang năng suất 100 kg mẻ.doc
ĐỒ ÁN - Tính toán thiết kế máy sấy khoai lang năng suất 100 kg mẻ.doc
 
Đồ án tốt nghiệp - Sấy bã mía, 9 điểm.doc
Đồ án tốt nghiệp - Sấy bã mía, 9 điểm.docĐồ án tốt nghiệp - Sấy bã mía, 9 điểm.doc
Đồ án tốt nghiệp - Sấy bã mía, 9 điểm.doc
 
Hoàn thiện quy trình sản xuất thanh long sấy bằng phương pháp sấy đối lưu.doc
Hoàn thiện quy trình sản xuất thanh long sấy bằng phương pháp sấy đối lưu.docHoàn thiện quy trình sản xuất thanh long sấy bằng phương pháp sấy đối lưu.doc
Hoàn thiện quy trình sản xuất thanh long sấy bằng phương pháp sấy đối lưu.doc
 
ĐỒ ÁN - Điều khiển lưu lượng không khí trong phòng sạch thông qua biến tần.doc
ĐỒ ÁN - Điều khiển lưu lượng không khí trong phòng sạch thông qua biến tần.docĐỒ ÁN - Điều khiển lưu lượng không khí trong phòng sạch thông qua biến tần.doc
ĐỒ ÁN - Điều khiển lưu lượng không khí trong phòng sạch thông qua biến tần.doc
 
ĐỒ ÁN - Tính toán thiết bị sấy nấm kểu sấy hầm, năng suất nhập liệu 650kgmẻ.doc
ĐỒ ÁN - Tính toán thiết bị sấy nấm kểu sấy hầm, năng suất nhập liệu 650kgmẻ.docĐỒ ÁN - Tính toán thiết bị sấy nấm kểu sấy hầm, năng suất nhập liệu 650kgmẻ.doc
ĐỒ ÁN - Tính toán thiết bị sấy nấm kểu sấy hầm, năng suất nhập liệu 650kgmẻ.doc
 
Thiết kế nhà máy sản xuất bia năng suất 91,8 triệu lít sản phẩm năm.docx
Thiết kế nhà máy sản xuất bia năng suất 91,8 triệu lít sản phẩm năm.docxThiết kế nhà máy sản xuất bia năng suất 91,8 triệu lít sản phẩm năm.docx
Thiết kế nhà máy sản xuất bia năng suất 91,8 triệu lít sản phẩm năm.docx
 
Tính toán thiết kế hệ thống sấy thùng quay sấy cà phê nhân theo năng suất nhậ...
Tính toán thiết kế hệ thống sấy thùng quay sấy cà phê nhân theo năng suất nhậ...Tính toán thiết kế hệ thống sấy thùng quay sấy cà phê nhân theo năng suất nhậ...
Tính toán thiết kế hệ thống sấy thùng quay sấy cà phê nhân theo năng suất nhậ...
 
Thiết kế hệ thống sấy thùng quay sấy bắp với năng suất 800 kgh.docx
Thiết kế hệ thống sấy thùng quay sấy bắp với năng suất 800 kgh.docxThiết kế hệ thống sấy thùng quay sấy bắp với năng suất 800 kgh.docx
Thiết kế hệ thống sấy thùng quay sấy bắp với năng suất 800 kgh.docx
 

Recently uploaded

SÁNG KIẾN “THIẾT KẾ VÀ SỬ DỤNG INFOGRAPHIC TRONG DẠY HỌC ĐỊA LÍ 11 (BỘ SÁCH K...
SÁNG KIẾN “THIẾT KẾ VÀ SỬ DỤNG INFOGRAPHIC TRONG DẠY HỌC ĐỊA LÍ 11 (BỘ SÁCH K...SÁNG KIẾN “THIẾT KẾ VÀ SỬ DỤNG INFOGRAPHIC TRONG DẠY HỌC ĐỊA LÍ 11 (BỘ SÁCH K...
SÁNG KIẾN “THIẾT KẾ VÀ SỬ DỤNG INFOGRAPHIC TRONG DẠY HỌC ĐỊA LÍ 11 (BỘ SÁCH K...Nguyen Thanh Tu Collection
 
Kiểm tra chạy trạm lí thuyết giữa kì giải phẫu sinh lí
Kiểm tra chạy trạm lí thuyết giữa kì giải phẫu sinh líKiểm tra chạy trạm lí thuyết giữa kì giải phẫu sinh lí
Kiểm tra chạy trạm lí thuyết giữa kì giải phẫu sinh líDr K-OGN
 
Sáng kiến Dạy học theo định hướng STEM một số chủ đề phần “vật sống”, Khoa họ...
Sáng kiến Dạy học theo định hướng STEM một số chủ đề phần “vật sống”, Khoa họ...Sáng kiến Dạy học theo định hướng STEM một số chủ đề phần “vật sống”, Khoa họ...
Sáng kiến Dạy học theo định hướng STEM một số chủ đề phần “vật sống”, Khoa họ...Nguyen Thanh Tu Collection
 
TỔNG HỢP ĐỀ THI CHÍNH THỨC KỲ THI TUYỂN SINH VÀO LỚP 10 THPT MÔN NGỮ VĂN NĂM ...
TỔNG HỢP ĐỀ THI CHÍNH THỨC KỲ THI TUYỂN SINH VÀO LỚP 10 THPT MÔN NGỮ VĂN NĂM ...TỔNG HỢP ĐỀ THI CHÍNH THỨC KỲ THI TUYỂN SINH VÀO LỚP 10 THPT MÔN NGỮ VĂN NĂM ...
TỔNG HỢP ĐỀ THI CHÍNH THỨC KỲ THI TUYỂN SINH VÀO LỚP 10 THPT MÔN NGỮ VĂN NĂM ...Nguyen Thanh Tu Collection
 
Sáng kiến “Sử dụng ứng dụng Quizizz nhằm nâng cao chất lượng ôn thi tốt nghiệ...
Sáng kiến “Sử dụng ứng dụng Quizizz nhằm nâng cao chất lượng ôn thi tốt nghiệ...Sáng kiến “Sử dụng ứng dụng Quizizz nhằm nâng cao chất lượng ôn thi tốt nghiệ...
Sáng kiến “Sử dụng ứng dụng Quizizz nhằm nâng cao chất lượng ôn thi tốt nghiệ...Nguyen Thanh Tu Collection
 
Chàm - Bệnh án (da liễu - bvdlct ctump) .pptx
Chàm - Bệnh án (da liễu - bvdlct ctump) .pptxChàm - Bệnh án (da liễu - bvdlct ctump) .pptx
Chàm - Bệnh án (da liễu - bvdlct ctump) .pptxendkay31
 
QUẢN LÝ HOẠT ĐỘNG GIÁO DỤC KỸ NĂNG SỐNG CHO HỌC SINH CÁC TRƯỜNG TRUNG HỌC CƠ ...
QUẢN LÝ HOẠT ĐỘNG GIÁO DỤC KỸ NĂNG SỐNG CHO HỌC SINH CÁC TRƯỜNG TRUNG HỌC CƠ ...QUẢN LÝ HOẠT ĐỘNG GIÁO DỤC KỸ NĂNG SỐNG CHO HỌC SINH CÁC TRƯỜNG TRUNG HỌC CƠ ...
QUẢN LÝ HOẠT ĐỘNG GIÁO DỤC KỸ NĂNG SỐNG CHO HỌC SINH CÁC TRƯỜNG TRUNG HỌC CƠ ...ThunTrn734461
 
30 ĐỀ PHÁT TRIỂN THEO CẤU TRÚC ĐỀ MINH HỌA BGD NGÀY 22-3-2024 KỲ THI TỐT NGHI...
30 ĐỀ PHÁT TRIỂN THEO CẤU TRÚC ĐỀ MINH HỌA BGD NGÀY 22-3-2024 KỲ THI TỐT NGHI...30 ĐỀ PHÁT TRIỂN THEO CẤU TRÚC ĐỀ MINH HỌA BGD NGÀY 22-3-2024 KỲ THI TỐT NGHI...
30 ĐỀ PHÁT TRIỂN THEO CẤU TRÚC ĐỀ MINH HỌA BGD NGÀY 22-3-2024 KỲ THI TỐT NGHI...Nguyen Thanh Tu Collection
 
BỘ ĐỀ PHÁT TRIỂN THEO CẤU TRÚC ĐỀ MINH HỌA BGD NGÀY 22-3-2024 KỲ THI TỐT NGHI...
BỘ ĐỀ PHÁT TRIỂN THEO CẤU TRÚC ĐỀ MINH HỌA BGD NGÀY 22-3-2024 KỲ THI TỐT NGHI...BỘ ĐỀ PHÁT TRIỂN THEO CẤU TRÚC ĐỀ MINH HỌA BGD NGÀY 22-3-2024 KỲ THI TỐT NGHI...
BỘ ĐỀ PHÁT TRIỂN THEO CẤU TRÚC ĐỀ MINH HỌA BGD NGÀY 22-3-2024 KỲ THI TỐT NGHI...Nguyen Thanh Tu Collection
 
NQA Lợi ích Từ ISO và ESG Tăng Trưởng và Bền Vững ver01.pdf
NQA Lợi ích Từ ISO và ESG Tăng Trưởng và Bền Vững ver01.pdfNQA Lợi ích Từ ISO và ESG Tăng Trưởng và Bền Vững ver01.pdf
NQA Lợi ích Từ ISO và ESG Tăng Trưởng và Bền Vững ver01.pdfNguyễn Đăng Quang
 
bài 5.1.docx Sinh học di truyền đại cương năm nhất của học sinh y đa khoa
bài 5.1.docx Sinh học di truyền đại cương năm nhất của học sinh y đa khoabài 5.1.docx Sinh học di truyền đại cương năm nhất của học sinh y đa khoa
bài 5.1.docx Sinh học di truyền đại cương năm nhất của học sinh y đa khoa2353020138
 
Sơ đồ tư duy môn sinh học bậc THPT.pdf
Sơ đồ tư duy môn sinh học bậc THPT.pdfSơ đồ tư duy môn sinh học bậc THPT.pdf
Sơ đồ tư duy môn sinh học bậc THPT.pdftohoanggiabao81
 
BỘ ĐỀ KIỂM TRA CUỐI KÌ 2 VẬT LÝ 11 - KẾT NỐI TRI THỨC - THEO CẤU TRÚC ĐỀ MIN...
BỘ ĐỀ KIỂM TRA CUỐI KÌ 2 VẬT LÝ 11 - KẾT NỐI TRI THỨC - THEO CẤU TRÚC ĐỀ MIN...BỘ ĐỀ KIỂM TRA CUỐI KÌ 2 VẬT LÝ 11 - KẾT NỐI TRI THỨC - THEO CẤU TRÚC ĐỀ MIN...
BỘ ĐỀ KIỂM TRA CUỐI KÌ 2 VẬT LÝ 11 - KẾT NỐI TRI THỨC - THEO CẤU TRÚC ĐỀ MIN...Nguyen Thanh Tu Collection
 
10 ĐỀ KIỂM TRA + 6 ĐỀ ÔN TẬP CUỐI KÌ 2 VẬT LÝ 11 - KẾT NỐI TRI THỨC - THEO C...
10 ĐỀ KIỂM TRA + 6 ĐỀ ÔN TẬP CUỐI KÌ 2 VẬT LÝ 11 - KẾT NỐI TRI THỨC - THEO C...10 ĐỀ KIỂM TRA + 6 ĐỀ ÔN TẬP CUỐI KÌ 2 VẬT LÝ 11 - KẾT NỐI TRI THỨC - THEO C...
10 ĐỀ KIỂM TRA + 6 ĐỀ ÔN TẬP CUỐI KÌ 2 VẬT LÝ 11 - KẾT NỐI TRI THỨC - THEO C...Nguyen Thanh Tu Collection
 
Chuong trinh dao tao Su pham Khoa hoc tu nhien, ma nganh - 7140247.pdf
Chuong trinh dao tao Su pham Khoa hoc tu nhien, ma nganh - 7140247.pdfChuong trinh dao tao Su pham Khoa hoc tu nhien, ma nganh - 7140247.pdf
Chuong trinh dao tao Su pham Khoa hoc tu nhien, ma nganh - 7140247.pdfhoangtuansinh1
 
Trích dẫn trắc nghiệm tư tưởng HCM5.docx
Trích dẫn trắc nghiệm tư tưởng HCM5.docxTrích dẫn trắc nghiệm tư tưởng HCM5.docx
Trích dẫn trắc nghiệm tư tưởng HCM5.docxnhungdt08102004
 
Thong bao 337-DHPY (24.4.2024) thi sat hach Ngoai ngu dap ung Chuan dau ra do...
Thong bao 337-DHPY (24.4.2024) thi sat hach Ngoai ngu dap ung Chuan dau ra do...Thong bao 337-DHPY (24.4.2024) thi sat hach Ngoai ngu dap ung Chuan dau ra do...
Thong bao 337-DHPY (24.4.2024) thi sat hach Ngoai ngu dap ung Chuan dau ra do...hoangtuansinh1
 
ôn tập lịch sử hhhhhhhhhhhhhhhhhhhhhhhhhh
ôn tập lịch sử hhhhhhhhhhhhhhhhhhhhhhhhhhôn tập lịch sử hhhhhhhhhhhhhhhhhhhhhhhhhh
ôn tập lịch sử hhhhhhhhhhhhhhhhhhhhhhhhhhvanhathvc
 
30 ĐỀ PHÁT TRIỂN THEO CẤU TRÚC ĐỀ MINH HỌA BGD NGÀY 22-3-2024 KỲ THI TỐT NGHI...
30 ĐỀ PHÁT TRIỂN THEO CẤU TRÚC ĐỀ MINH HỌA BGD NGÀY 22-3-2024 KỲ THI TỐT NGHI...30 ĐỀ PHÁT TRIỂN THEO CẤU TRÚC ĐỀ MINH HỌA BGD NGÀY 22-3-2024 KỲ THI TỐT NGHI...
30 ĐỀ PHÁT TRIỂN THEO CẤU TRÚC ĐỀ MINH HỌA BGD NGÀY 22-3-2024 KỲ THI TỐT NGHI...Nguyen Thanh Tu Collection
 

Recently uploaded (19)

SÁNG KIẾN “THIẾT KẾ VÀ SỬ DỤNG INFOGRAPHIC TRONG DẠY HỌC ĐỊA LÍ 11 (BỘ SÁCH K...
SÁNG KIẾN “THIẾT KẾ VÀ SỬ DỤNG INFOGRAPHIC TRONG DẠY HỌC ĐỊA LÍ 11 (BỘ SÁCH K...SÁNG KIẾN “THIẾT KẾ VÀ SỬ DỤNG INFOGRAPHIC TRONG DẠY HỌC ĐỊA LÍ 11 (BỘ SÁCH K...
SÁNG KIẾN “THIẾT KẾ VÀ SỬ DỤNG INFOGRAPHIC TRONG DẠY HỌC ĐỊA LÍ 11 (BỘ SÁCH K...
 
Kiểm tra chạy trạm lí thuyết giữa kì giải phẫu sinh lí
Kiểm tra chạy trạm lí thuyết giữa kì giải phẫu sinh líKiểm tra chạy trạm lí thuyết giữa kì giải phẫu sinh lí
Kiểm tra chạy trạm lí thuyết giữa kì giải phẫu sinh lí
 
Sáng kiến Dạy học theo định hướng STEM một số chủ đề phần “vật sống”, Khoa họ...
Sáng kiến Dạy học theo định hướng STEM một số chủ đề phần “vật sống”, Khoa họ...Sáng kiến Dạy học theo định hướng STEM một số chủ đề phần “vật sống”, Khoa họ...
Sáng kiến Dạy học theo định hướng STEM một số chủ đề phần “vật sống”, Khoa họ...
 
TỔNG HỢP ĐỀ THI CHÍNH THỨC KỲ THI TUYỂN SINH VÀO LỚP 10 THPT MÔN NGỮ VĂN NĂM ...
TỔNG HỢP ĐỀ THI CHÍNH THỨC KỲ THI TUYỂN SINH VÀO LỚP 10 THPT MÔN NGỮ VĂN NĂM ...TỔNG HỢP ĐỀ THI CHÍNH THỨC KỲ THI TUYỂN SINH VÀO LỚP 10 THPT MÔN NGỮ VĂN NĂM ...
TỔNG HỢP ĐỀ THI CHÍNH THỨC KỲ THI TUYỂN SINH VÀO LỚP 10 THPT MÔN NGỮ VĂN NĂM ...
 
Sáng kiến “Sử dụng ứng dụng Quizizz nhằm nâng cao chất lượng ôn thi tốt nghiệ...
Sáng kiến “Sử dụng ứng dụng Quizizz nhằm nâng cao chất lượng ôn thi tốt nghiệ...Sáng kiến “Sử dụng ứng dụng Quizizz nhằm nâng cao chất lượng ôn thi tốt nghiệ...
Sáng kiến “Sử dụng ứng dụng Quizizz nhằm nâng cao chất lượng ôn thi tốt nghiệ...
 
Chàm - Bệnh án (da liễu - bvdlct ctump) .pptx
Chàm - Bệnh án (da liễu - bvdlct ctump) .pptxChàm - Bệnh án (da liễu - bvdlct ctump) .pptx
Chàm - Bệnh án (da liễu - bvdlct ctump) .pptx
 
QUẢN LÝ HOẠT ĐỘNG GIÁO DỤC KỸ NĂNG SỐNG CHO HỌC SINH CÁC TRƯỜNG TRUNG HỌC CƠ ...
QUẢN LÝ HOẠT ĐỘNG GIÁO DỤC KỸ NĂNG SỐNG CHO HỌC SINH CÁC TRƯỜNG TRUNG HỌC CƠ ...QUẢN LÝ HOẠT ĐỘNG GIÁO DỤC KỸ NĂNG SỐNG CHO HỌC SINH CÁC TRƯỜNG TRUNG HỌC CƠ ...
QUẢN LÝ HOẠT ĐỘNG GIÁO DỤC KỸ NĂNG SỐNG CHO HỌC SINH CÁC TRƯỜNG TRUNG HỌC CƠ ...
 
30 ĐỀ PHÁT TRIỂN THEO CẤU TRÚC ĐỀ MINH HỌA BGD NGÀY 22-3-2024 KỲ THI TỐT NGHI...
30 ĐỀ PHÁT TRIỂN THEO CẤU TRÚC ĐỀ MINH HỌA BGD NGÀY 22-3-2024 KỲ THI TỐT NGHI...30 ĐỀ PHÁT TRIỂN THEO CẤU TRÚC ĐỀ MINH HỌA BGD NGÀY 22-3-2024 KỲ THI TỐT NGHI...
30 ĐỀ PHÁT TRIỂN THEO CẤU TRÚC ĐỀ MINH HỌA BGD NGÀY 22-3-2024 KỲ THI TỐT NGHI...
 
BỘ ĐỀ PHÁT TRIỂN THEO CẤU TRÚC ĐỀ MINH HỌA BGD NGÀY 22-3-2024 KỲ THI TỐT NGHI...
BỘ ĐỀ PHÁT TRIỂN THEO CẤU TRÚC ĐỀ MINH HỌA BGD NGÀY 22-3-2024 KỲ THI TỐT NGHI...BỘ ĐỀ PHÁT TRIỂN THEO CẤU TRÚC ĐỀ MINH HỌA BGD NGÀY 22-3-2024 KỲ THI TỐT NGHI...
BỘ ĐỀ PHÁT TRIỂN THEO CẤU TRÚC ĐỀ MINH HỌA BGD NGÀY 22-3-2024 KỲ THI TỐT NGHI...
 
NQA Lợi ích Từ ISO và ESG Tăng Trưởng và Bền Vững ver01.pdf
NQA Lợi ích Từ ISO và ESG Tăng Trưởng và Bền Vững ver01.pdfNQA Lợi ích Từ ISO và ESG Tăng Trưởng và Bền Vững ver01.pdf
NQA Lợi ích Từ ISO và ESG Tăng Trưởng và Bền Vững ver01.pdf
 
bài 5.1.docx Sinh học di truyền đại cương năm nhất của học sinh y đa khoa
bài 5.1.docx Sinh học di truyền đại cương năm nhất của học sinh y đa khoabài 5.1.docx Sinh học di truyền đại cương năm nhất của học sinh y đa khoa
bài 5.1.docx Sinh học di truyền đại cương năm nhất của học sinh y đa khoa
 
Sơ đồ tư duy môn sinh học bậc THPT.pdf
Sơ đồ tư duy môn sinh học bậc THPT.pdfSơ đồ tư duy môn sinh học bậc THPT.pdf
Sơ đồ tư duy môn sinh học bậc THPT.pdf
 
BỘ ĐỀ KIỂM TRA CUỐI KÌ 2 VẬT LÝ 11 - KẾT NỐI TRI THỨC - THEO CẤU TRÚC ĐỀ MIN...
BỘ ĐỀ KIỂM TRA CUỐI KÌ 2 VẬT LÝ 11 - KẾT NỐI TRI THỨC - THEO CẤU TRÚC ĐỀ MIN...BỘ ĐỀ KIỂM TRA CUỐI KÌ 2 VẬT LÝ 11 - KẾT NỐI TRI THỨC - THEO CẤU TRÚC ĐỀ MIN...
BỘ ĐỀ KIỂM TRA CUỐI KÌ 2 VẬT LÝ 11 - KẾT NỐI TRI THỨC - THEO CẤU TRÚC ĐỀ MIN...
 
10 ĐỀ KIỂM TRA + 6 ĐỀ ÔN TẬP CUỐI KÌ 2 VẬT LÝ 11 - KẾT NỐI TRI THỨC - THEO C...
10 ĐỀ KIỂM TRA + 6 ĐỀ ÔN TẬP CUỐI KÌ 2 VẬT LÝ 11 - KẾT NỐI TRI THỨC - THEO C...10 ĐỀ KIỂM TRA + 6 ĐỀ ÔN TẬP CUỐI KÌ 2 VẬT LÝ 11 - KẾT NỐI TRI THỨC - THEO C...
10 ĐỀ KIỂM TRA + 6 ĐỀ ÔN TẬP CUỐI KÌ 2 VẬT LÝ 11 - KẾT NỐI TRI THỨC - THEO C...
 
Chuong trinh dao tao Su pham Khoa hoc tu nhien, ma nganh - 7140247.pdf
Chuong trinh dao tao Su pham Khoa hoc tu nhien, ma nganh - 7140247.pdfChuong trinh dao tao Su pham Khoa hoc tu nhien, ma nganh - 7140247.pdf
Chuong trinh dao tao Su pham Khoa hoc tu nhien, ma nganh - 7140247.pdf
 
Trích dẫn trắc nghiệm tư tưởng HCM5.docx
Trích dẫn trắc nghiệm tư tưởng HCM5.docxTrích dẫn trắc nghiệm tư tưởng HCM5.docx
Trích dẫn trắc nghiệm tư tưởng HCM5.docx
 
Thong bao 337-DHPY (24.4.2024) thi sat hach Ngoai ngu dap ung Chuan dau ra do...
Thong bao 337-DHPY (24.4.2024) thi sat hach Ngoai ngu dap ung Chuan dau ra do...Thong bao 337-DHPY (24.4.2024) thi sat hach Ngoai ngu dap ung Chuan dau ra do...
Thong bao 337-DHPY (24.4.2024) thi sat hach Ngoai ngu dap ung Chuan dau ra do...
 
ôn tập lịch sử hhhhhhhhhhhhhhhhhhhhhhhhhh
ôn tập lịch sử hhhhhhhhhhhhhhhhhhhhhhhhhhôn tập lịch sử hhhhhhhhhhhhhhhhhhhhhhhhhh
ôn tập lịch sử hhhhhhhhhhhhhhhhhhhhhhhhhh
 
30 ĐỀ PHÁT TRIỂN THEO CẤU TRÚC ĐỀ MINH HỌA BGD NGÀY 22-3-2024 KỲ THI TỐT NGHI...
30 ĐỀ PHÁT TRIỂN THEO CẤU TRÚC ĐỀ MINH HỌA BGD NGÀY 22-3-2024 KỲ THI TỐT NGHI...30 ĐỀ PHÁT TRIỂN THEO CẤU TRÚC ĐỀ MINH HỌA BGD NGÀY 22-3-2024 KỲ THI TỐT NGHI...
30 ĐỀ PHÁT TRIỂN THEO CẤU TRÚC ĐỀ MINH HỌA BGD NGÀY 22-3-2024 KỲ THI TỐT NGHI...
 

Luận văn thạc sĩ toán học - Một số lớp phương trình diophantine.doc

  • 1. Viết thuê đề tài giá rẻ trọn gói - KB Zalo/Tele : 0973.287.149 Luanvanmaster.com – Cần Kham Thảo - Kết bạn Zalo/Tele : 0973.287.149 ĐẠI HỌC THÁI NGUYÊN TRƯỜNG ĐẠI HỌC KHOA HỌC BÙI HỮU MÊN MỘT SỐ LỚP PHƯƠNG TRÌNH DIOPHANTINE LUẬN VĂN THẠC SĨ TOÁN HỌC Thái Nguyên
  • 2. Viết thuê đề tài giá rẻ trọn gói - KB Zalo/Tele : 0973.287.149 Luanvanmaster.com – Cần Kham Thảo - Kết bạn Zalo/Tele : 0973.287.149 ĐẠI HỌC THÁI NGUYÊN TRƯỜNG ĐẠI HỌC KHOA HỌC BÙI HỮU MÊN MỘT SỐ LỚP PHƯƠNG TRÌNH DIOPHANTINE LUẬN VĂN THẠC SĨ TOÁN HỌC Chuyên ngành:Phương pháp Toán sơ cấp Mã số: 60 46 01 13 NGƯỜI HƯỚNG DẪN KHOA HỌC GS.TSKH. ĐẶNG HÙNG THẮNG Thái Nguyên
  • 3. Viết thuê đề tài giá rẻ trọn gói - KB Zalo/Tele : 0973.287.149 Luanvanmaster.com – Cần Kham Thảo - Kết bạn Zalo/Tele : 0973.287.149 3 Mục lục Danh sách kí hiệu 4 Mở đầu 5 Chương 1. Phương trình Diophantine tuyến tính 7 1.1 Phương trình bậc nhất hai ẩn . . . . . . . . . . . . . . . . . . . . . . . 8 1.2 Phương trình bậc nhất nhiều ẩn . . . . . . . . . . . . . . . . . . . . . . 15 Chương 2. Một số phương trình Diophantine phi tuyến 23 2.1 Phương trình Pell loại 1 . . . . . . . . . . . . . . . . . . . . . . . . . . 23 2.2 Phương trình Pell loại 2 . . . . . . . . . . . . . . . . . . . . . . . . . . 30 2.3 Phương trình Pythagoras . . . . . . . . . . . . . . . . . . . . . . . . . 38 Chương 3. Liên phân số và ứng dụng trong phương trình Diophantine 45 3.1 Liên phân số hữu hạn . . . . . . . . . . . . . . . . . . . . . . . . . . . 45 3.2 Liên phân số vô hạn . . . . . . . . . . . . . . . . . . . . . . . . . . . . 49 3.3 Liên phân số vô hạn tuần hoàn . . . . . . . . . . . . . . . . . . . . . . 50 3.4 Áp dụng vào phương trình Diophante . . . . . . . . . . . . . . . . . . 56 3.4.1 Phương trình bậc nhất hai ẩn Ax + By = C . . . . . . . . . . . . 56 3.4.2 Phương trình x 2 dy 2 = 1 . . . . . . . . . . . . . . . . . . . 57 Kết luận 62 Tài liệu tham khảo 63
  • 4. Viết thuê đề tài giá rẻ trọn gói - KB Zalo/Tele : 0973.287.149 Luanvanmaster.com – Cần Kham Thảo - Kết bạn Zalo/Tele : 0973.287.149 4 Danh sách kí hiệu N Z Q R C Fp K[X] dxe deg P(X) mod p gcd(P(X); Q(X)) tập hợp các số tự nhiên vành các số nguyên trường các số hữu tỷ trường các số thực trường các số phức trường có p phần tử vành đa thức với hệ số trên trường K trần của số x bậc của đa thức P(X) modulo p ước chung lớn nhất của hai đa thức P(X) và Q(X)
  • 5. Viết thuê đề tài giá rẻ trọn gói - KB Zalo/Tele : 0973.287.149 Luanvanmaster.com – Cần Kham Thảo - Kết bạn Zalo/Tele : 0973.287.149 5 Mở đầu Phương trình Diophantine là một chủ đề lớn của Lý thuyết số, chứa đựng nhiều lý thuyết toán học sâu sắc, gắn liền với nhiều tên tuổi của nhiều nhà toán học xuất sắc. Mục tiêu của đề tài luận văn là: Tìm hiểu một số lớp phương trình Diophantine như: phương trình Diophantine tuyến tính; một số phương trình Diophantine phi tuyến (phương trình Pell, phương trình Pell mở rộng, phương trình Pythagoras Fermat). Liên phân số và ứng dụng trong phương trình Diophantine. Về mặt ứng dụng, luận văn sẽ áp dụng lý thuyết để soi sáng những bài toán số học ở phổ thông, hệ thống hóa, tổng quát hóa và sáng tác ra những bài toán số học mới. Luận văn sẽ cố gắng trở thành một tài liệu tham khảo tốt, thiết thực phục vụ cho việc giảng dạy, nhất là việc giảng dạy và bồi dưỡng học sinh giỏi. Ngoài ra thông qua việc viết luận văn, tác giả luận văn có cơ hội mở rộng nâng cao hiểu biết về toán sơ cấp nói chung và số học nói riêng, hình thành các kỹ năng chứng minh các định lí số học và giải các bài toán số học, phục vụ tốt cho việc giảng dạy môn Toán ở trường phổ thông. Nội dung của luận văn được trình bày trong ba chương như sau: Chương 1. Phương trình Diophantine tuyến tính. Trong chương này chúng tôi trình bày về phương trình bậc nhất hai ẩn, nhiều ẩn, và một số bài toán chọn lọc. Chương 2. Một số phương trình Diophantine phi tuyến. Trong chương này chúng tôi trình bày nội dung chính về các phương trình Pell loại 1, phương trình Pell loại , và phương trình Pythagoras. Chương 3. Liên phân số và ứng dụng trong phương trình Diophantine. Trong
  • 6. Viết thuê đề tài giá rẻ trọn gói - KB Zalo/Tele : 0973.287.149 Luanvanmaster.com – Cần Kham Thảo - Kết bạn Zalo/Tele : 0973.287.149 6 chương này chúng tôi trình bày một cách ngắn gọn các sự kiện về liên phân số, đặc biệt là các ứng dụng của chúng để giải phương trình Pell. Luận văn này được thực hiện tại Trường Đại học Khoa học - Đại học Thái Nguyên và hoàn thành với sự hướng dẫn của GS.TSKH. Đặng Hùng Thắng (Trường ĐHKHTN - ĐHQG Hà Nội). Tác giả xin được bày tỏ lòng biết ơn chân thành và sâu sắc tới người hướng dẫn khoa học của mình, người đã đặt vấn đề nghiên cứu, dành nhiều thời gian hướng dẫn và tận tình giải đáp những thắc mắc của tác giả trong suốt quá trình làm luận văn. Tác giả xin trân trọng cảm ơn Ban Giám hiệu Trường Đại học Khoa học - Đại học Thái Nguyên, Ban Chủ nhiệm Khoa Toán–Tin, cùng các giảng viên đã tham gia giảng dạy, đã tạo mọi điều kiện tốt nhất để tác giả học tập và nghiên cứu. Tác giả muốn gửi những lời cảm ơn tốt đẹp nhất tới tập thể lớp Cao học Toán khóa 9 (2015-2017) đã động viên và giúp đỡ tác giả rất nhiều trong suốt quá trình học tập. Nhân dịp này, tác giả cũng xin chân thành cảm ơn Sở Giáo dục và Đào tạo Hải Phòng, Ban Giám hiệu và các đồng nghiệp ở Trường THPT Thái Phiên đã tạo điều kiện cho tác giả hoàn thành tốt nhiệm vụ học tập và công tác của mình. Cuối cùng, tác giả muốn dành những lời cảm ơn đặc biệt nhất đến đại gia đình vì những động viên và chia sẻ những khó khăn để tác giả hoàn thành luận văn này. Thái Nguyên, ngày 10 tháng 11 năm 2017 Tác giả Bùi Hữu Mên
  • 7. Viết thuê đề tài giá rẻ trọn gói - KB Zalo/Tele : 0973.287.149 Luanvanmaster.com – Cần Kham Thảo - Kết bạn Zalo/Tele : 0973.287.149 7 Chương 1 Phương trình Diophantine tuyến tính Phương trình Diophantine là một trong những chủ đề sâu sắc và rất rộng của Lý thuyết số. Mục đích của chương này là nghiên cứu về phương trình Diophantine bậc nhất hai và nhiều ẩn. Như một minh họa cho lý thuyết, các ví dụ là các bài toán trích từ các đề thi sẽ được trình bày. Đặc tính của các phương trình Diophantine là chúng có một hay nhiều ẩn số mà mọi hệ số đều là số nguyên và chỉ yêu cầu tìm các nghiệm nguyên (hoặc nguyên dương). Nhà toán học nổi tiếng thời cổ đại Diophantine đã có công lớn vì những nghiên cứu tiên phong về chúng. Với một phương trình Diophantine cho trước ta có thể đặt ra các câu hỏi sau đây (xếp theo thứ tự từ dễ đến khó): Câu hỏi 1. Nó có nghiệm nguyên hay không ? Câu hỏi 2. Nó có một số hữu hạn nghiệm hay có vô số nghiệm? Câu hỏi 3. Hãy tìm tất cả các nghiệm của nó. Chẳng hạn, ta hãy xét phương trình Diophantine x n + y n = z n trong đó n là số nguyên dương lớn hơn hay bằng 2. Với n = 2 phương trình trên có vô số nghiệm và ta có thể tìm được tường minh tất cả các nghiệm của nó. Với n > 2, nhà toán học thiên tài của thế kỷ 17 Pierre de Fermat khẳng định rằng
  • 8. Viết thuê đề tài giá rẻ trọn gói - KB Zalo/Tele : 0973.287.149 Luanvanmaster.com – Cần Kham Thảo - Kết bạn Zalo/Tele : 0973.287.149 8 phương trình trên không có nghiệm nguyên dương. Kết luận này ngày nay được mang tên là Định lí lớn Fermat hay Định lí cuối cùng của Fermat. Người ta đã không tìm thấy dấu vết của chứng minh khẳng định trên của Fermat mà chỉ thấy một ghi chú của Fermat bên lề cuốn sách “Số học” của Diophantine: “Tôi đã tìm được một chứng minh thật là tuyệt vời nhưng vì lề sách ở đây quá hẹp nên không thể viết ra”. Năm 1983, nhà toán học 29 tuổi người Đức là Faltings đã chứng minh thành công một giả thuyết của Mordell trong lĩnh vực Hình học đại số rồi từ đó suy ra rằng phương trình x n + y n = z n với n > 2 chỉ có một số hữu hạn nghiệm nguyên. Với thành tựu này Faltings đã nhận được Giải thưởng Fields (giải thưởng quốc tế cao nhất dành cho các nhà toán học không quá 40 tuổi). Năm 1993 nhà toán học người Anh là Andrew Wiles đã công bố phép chứng minh của Định lí lớn Fermat. Đây là một câu chuyện lớn của Toán học, có thể tham khảo trong Amir D. Aczel [1]. Với sự ra đời của máy tính, người ta cũng đặt câu hỏi: Có tồn tại chăng một thuật toán để với mọi phương trình Diophantine cho trước nhờ đó có thể khẳng định được rằng phương trình này có nghiệm nguyên hay không. Tiếc thay câu trả lời lại là: không có một thuật toán như vậy (Định lí Machiakevich). 1.1 Phương trình bậc nhất hai ẩn Phương trình Diophantine đơn giản nhất là phương trình bậc nhất hai ẩn ax + by = c (1.1) trong đó a, b, c là những số nguyên cho trước khác 0. Vấn đề đặt ra là với điều kiện nào của a, b, c thì phương trình (1.1) có nghiệm và nếu có thì cách tìm nghiệm thế nào. Định lí 1.1.1. Điều kiện cần và đủ để phương trình (1.1) có nghiệm nguyên là
  • 9. Viết thuê đề tài giá rẻ trọn gói - KB Zalo/Tele : 0973.287.149 Luanvanmaster.com – Cần Kham Thảo - Kết bạn Zalo/Tele : 0973.287.149 9 (a; b) là ước của c. Chứng minh. Điều kiện cần. Giả sử (x0; y0) là một nghiệm nguyên của (1.1). Khi đó ax0 + by0 = c. Nếu d = (a; b) thì rõ ràng d j c. Điều kiện đủ. Giả sử d = (a; b) và d j c. Ta có a = da1, b = db1, c = dc1 trong đó (a1; b1) = 1. Phương trình (1.1) tương đương với a1x + b1y = c1. Xét a1 số fb1kg với k = 0; 1; 2; : : : ; a1 1. Vì (a1; b1) = 1 nên các số này khi chia cho a1 sẽ cho ta các số dư khác nhau. Vậy tại k0, 0 k0 a1 1 sao cho b1k0 = c1 (mod a1). Điều này có nghĩa là: c1 b1k0 = a1l0 với l0 2 Z hay c1 = a1l0 + b1k0: Vậy (l0; k0) là một nghiệm của phương trình (1.1). Phép chứng minh định lí được hoàn thành. Tiếp theo ta hãy đi tìm tất cả các nghiệm của phương trình (1.1) Định lí 1.1.2. Nếu (x0; y0) là một nghiệm nguyên của (1.1) thì nó có vô số nghiệm nguyên và nghiệm nguyên (x; y) của nó được cho bởi công thức 8 x= x0 + b t; d (1.2) > y= y 0 a t; < d > : trong đó t 2 Z và d = (a; b): Chứng minh. Trước hết ta kiểm tra mọi cặp số (x; y) cho bởi công thức (1.2) là nghiệm. Thật vậy ax + by = ax0 + by0 = c: Đảo lại, giả sử (x1; y1) là một nghiệm của (1.1) tức là ax1 + by1 = c. Trừ đẳng thức này vào đẳng thức ax0 + by0 = c ta thu được a(x1 x0) = b(y0 y1): (1.3)
  • 10. Viết thuê đề tài giá rẻ trọn gói - KB Zalo/Tele : 0973.287.149 Luanvanmaster.com – Cần Kham Thảo - Kết bạn Zalo/Tele : 0973.287.149 10 Vì d = (a; b) nên a = a1d, b = b1d với (a1; b1) = 1. Thay vào (1.3) ta được a1(x1 x0) = b1(y0 y1). Vì (a1; b1) = 1 nên y0 y1 = ta1 và x1 x0 = tb1. Vậy y1 = y0 ta1 = y0 at và x1 = x0 +tb1 = x0 +bt : d d Phép chứng minh được kết thúc. Thuật toán tìm nghiệm của phương trình Diophantine bậc nhất. Từ Định lí 1.1.2 ta thấy rằng để tìm tất cả các nghiệm của (1.1) ta chỉ cần tìm một nghiệm (x0; y0) nào đó của nó. Ta gọi một nghiệm cụ thể như thế là một nghiệm riêng còn công thức (1.2) được gọi là nghiệm tổng quát. Sau đây ta sẽ trình bày một thuật toán cho phép xác định khá nhanh một nghiệm riêng của (1.1). Giả sử q0; q1; : : : là một dãy các số nguyên dương. Với mỗi i 0 ta kí hiệu [q0; q1; : : : ; qi] là phân số sau đây 1 [q0; q1; : : : ; qi] = q0 + : 1 q1 + 1 q2 + + 1 q i 1 +q i Bằng phương pháp quy nạp có thể dễ dàng chứng minh được bổ đề sau: Bổ đề 1.1.3. Giả sử fhng, fkng là hai dãy số nguyên được xác định như sau: h 2 = 0; h 1 = 1; h1 = qihi 1 + hi 2; i 0; k0 = 1; k1 = q1; ki = qihki 1 + ki 2; i 2: Khi đó với mọi i 1 ta có: (a) hiki 1 hi 1ki = ( 1) i 1 ; (b) [q0; q1 ; : : : ; qi] = h i . k i
  • 11. Viết thuê đề tài giá rẻ trọn gói - KB Zalo/Tele : 0973.287.149 Luanvanmaster.com – Cần Kham Thảo - Kết bạn Zalo/Tele : 0973.287.149 11 Bây giờ cho hai số dương a, b với a > b. Ta hãy viết thuật toán Euclid tìm ước chung lớn nhất của a và b. a = bq0 + r1 b = r1q1 + r2 : : : (1.4) r n 1 = r n q n 1 + r n+1 r n = r n+1 q n : Từ hệ (1.4) ta suy ra a = [q0; q1; : : : ; qn] b a = h n : b kn Từ (a) ta suy ra (hn; kn) = 1. Do đó nếu (a; b) = 1 thì a = hn và b = kn. Vậy thì akn 1 bhn 1 = ( 1) n 1 : Thành thử tồn tại x0 2 fkn 1g và y0 2 fhn 1g sao cho ax0 + by0 = 1: Ta thử từng trường hợp (nhiều nhất là bốn phép thử) để xác định x0, y0. Như vậy để giải phương trình (1.1) ta sẽ tiến hành lần lượt các bước sau đây. Bước 1. Tìm d = (a; b) sau đó chia hai vế cho d để được một phương trình tương đương a1x + b1y = c1 , ở đó a = da1; b = db1; c = dc1; (a1; b1) = 1: Bước 2. Viết thuật toán Euclid cho hai số ja1j và jb1j. Giả sử ja1j > jb1j. ja1j = jb1jq0 + r1
  • 12. Viết thuê đề tài giá rẻ trọn gói - KB Zalo/Tele : 0973.287.149 Luanvanmaster.com – Cần Kham Thảo - Kết bạn Zalo/Tele : 0973.287.149 12 jb1j = r1q1 + r2 : : : r n 1 = r n q n 1 + r n+1 r n = r n+1 q n : Bước 3. Tính [q0; q1; : : : ; qn 1] = h . k Bước 4. Lấy nghiệm riêng (x0 0 ; y0 0 ) của phương trình a1x + b1y = 1 thoả mãn điều kiện jx0 0 j = k, jy0 0 j = h. Ta xác định dấu của x0 0 và y0 0 bằng cách thử. Bước 5. Ta có x0 = c1x0 0 , y0 = c1y0 0 là nghiệm riêng của phương trình (1.1). Khi đó nghiệm tổng quát là x = x0 + b1t; y = y0 + a1t; với t 2 Z: Ví dụ 1.1.4. Giải phương trình Diophantine 342x 123y = 15: Lời giải. Ta sẽ làm lần lượt theo các bước như trên. Bước 1. Ứớc chung lớn nhất của 342 và 123 là 3. phương trình đã cho tương đương với 114x 41y = 5. Bước 2. Ta viết thuật toán Euclid cho 114 và 41. 114= 41 2+32 41= 32 1+9 32=9 3+5 9=5 1+4 5=4 1+1 4=1 4:
  • 13. Viết thuê đề tài giá rẻ trọn gói - KB Zalo/Tele : 0973.287.149 Luanvanmaster.com – Cần Kham Thảo - Kết bạn Zalo/Tele : 0973.287.149 Bước 3. Ta biểu diễn theo liên phân số h = 2 + Như vậy h = 25, k = 9. 13 1 = 25: 19 1+ 1 3 + 1 1 + 1 Bước 4. Lấy nghiệm riêng x0 0 , y0 0 của phương trình 114x 41y = 1 thoả mãn điều j 0j j 0j 0 0 kiện x 0 = 9 và y 0 = 25: Bằng cách thử ta tìm được x 0 = 9, y 0 = 25. Bước 5. Nghiệm riêng x0, y0 của phương trình đã cho là x0 = 9 5 = 45, y0 = 25 5 = 125 và nghiệm tổng quát là x = 45 + 41t; y = 125 + 114t; với t 2 Z: Nhận xét 1.1.5. 1. Nếu a j c ta có thể lấy nghiệm riêng x 0 =c ; y0 = 0: a 2. Xét trường hợp (a; b) = 1. Theo Định lí Euler ta có a j(b) 1 = kb với k 2 Z: Vậy ca j(b) bkc = c. Do đó x0 = ca j(b) 1; y0 = kc là một nghiệm riêng của phương trình đang xét.
  • 14. Viết thuê đề tài giá rẻ trọn gói - KB Zalo/Tele : 0973.287.149 Luanvanmaster.com – Cần Kham Thảo - Kết bạn Zalo/Tele : 0973.287.149 14 Một số bài toán chọn lọc Bài toán 1.1.6. Cho hai số nguyên dương a, b. Chứng minh rằng (a; b) = 1 khi và chỉ khi tồn tại các số nguyên dương u, v sao cho au bv = 1: Lời giải. Điều kiện đủ là hiển nhiên. Đảo lại, giả sử (a; b) = 1. Theo Định lí 1.1.1, tồn tại các số nguyên x0, y0 để cho ax0 + by0 = 1. Đặt 8 >u = x0 + bt; trong đó t < v= at y0; 2 Z > : x0 y0 t > ; t > : b b Khi đó u và v là các số nguyên dương và au bv = ax0 + by0 = 1. Bài toán 1.1.7. Giả sử (l; m) = 1 và a l = b m , trong đó a; b; l; m 2 N . Khi đó tồn tại n để a = nm , b = nl : Lời giải. Theo Bài toán 1.1.6 tồn tại các số r; s 2 N để lr ms = 1. Ta có alr bmr br m a = alr ms = : = = ams ams a s p b r p b r 2 N . m m Suy ra a = a s . Vì a là một số hữu tỉ nên nó phải là số nguyên. Vậy n = as Suy ra a = n m . Từ đó b m = n l = n ml , nên b = n l . Bài toán được chứng minh xong. Bài toán 1.1.8. Cho a 2 N . Hãy tìm g = (am 1; an 1): Lời giải. Trước hết xét trường hợp (m; n) = 1: Vì (a 1) j am 1; (a 1)an 1 nên a 1 là ước của g. Đảo lại, ta sẽ chứng minh g cũng là ước của a 1. Theo Bài toán 1.1.6 tồn tại các số u; v 2 N sao cho mu nv = 1:
  • 15. Viết thuê đề tài giá rẻ trọn gói - KB Zalo/Tele : 0973.287.149 Luanvanmaster.com – Cần Kham Thảo - Kết bạn Zalo/Tele : 0973.287.149 15 Vì g j a m 1, g j a n 1 nên cũng có g j a mu 1, g j a nv 1. Suy ra g j amu anv = anv(amu nv 1) = anv (a 1): Mặt khác, dễ thấy (g; a) = 1. Vậy g j (a 1): Như vậy nếu (m; n) = 1 thì (a m 1; a n 1) = a 1. Với m, n bất kì, giả sử d = (m; n): Khi ấy m = dm1, n = dn1 và (m1; n1) = 1: Ta có (am 1; an 1) = ((ad )m 1 1; (ad )n 1 1) = ad 1: Tóm lại ta có công thức (a m 1; a n 1) = a(m; n) 1: Bài toán 1.1.9. Cho (a; b) = 1 trong đó a; b 2 N . Tìm giá trị c 2 N lớn nhất để phương trình ax + by = c không có nghiệm nguyên dương. Lời giải. Ta chứng minh rằng c = ab là giá trị cần tìm. Giả sử c > ab. Xét b số a; 2a; : : : ; ba. Vì (a; b) = 1 nên các số này khi chia cho b sẽ cho các số dư khác nhau. Vậy tồn tại số k, 1 k b, sao cho ka c (mod b). Suy ra c ka = lb trong đó l 2 Z. Nếu c > ab thì c > ka, do đó lb > 0 hay l 2 N . Như vậy (k; l) là nghiệm nguyên dương của phương trình ax + by = c: Mặt khác, giả sử (x0; y0) là nghiệm nguyên dương của phương trình ax + by = ab. Khi đó ax0 = ab by0 = b(a y0). Vì (a; b) = 1 nên từ đó suy ra x0 .. . b. Tương tự, y0 .. . a. Như vậy x0 b, y0 a, do đó ab = ax0 + by0 2ab. Vô lý. Phép chứng minh được kết thúc. 1.2 Phương trình bậc nhất nhiều ẩn Trong mục này ta mở rộng kết quả của mục trước bằng cách xét phương trình Diophantine bậc nhất n ẩn a1x1 + a2x2 + : : : + anxn = c: (1.5)
  • 16. Viết thuê đề tài giá rẻ trọn gói - KB Zalo/Tele : 0973.287.149 Luanvanmaster.com – Cần Kham Thảo - Kết bạn Zalo/Tele : 0973.287.149 16 trong đó a1; a2; : : : ; an và c là các số nguyên cho trước, n 2. Đối với bất kỳ một phương trình nào, câu hỏi đầu tiên là, trong những tình huống nào của hệ số, ta có thể khẳng định về tính tồn tại nghiệm của nó. Về sự tồn tại nghiệm của phương trình Diophantine bậc nhất n ẩn này ta có định lí sau đây. Định lí 1.2.1. Điều kiện cần và đủ để phương trình (1.5) có nghiệm là (a1; a2; : : : ; an) j c: Chứng minh. Điều kiện cần là hiển nhiên. Ta sẽ chứng minh điều kiện đủ bằng phương pháp quy nạp, với n = 1 điều khẳng định là đúng do Định lí 1.1.1. Giả sử định lí đúng với n = 1, ta chứng minh nó đúng với n. Kí hiệu ai c bi = ; c1 = ; ở đó d = (a1 ; a2; : : : ; an): d d Phương trình (1.5) tương đương với b1x1+ b2x2 + : : : + bnxn = c1: (1.6) trong đó (b1; b2; : : : ; bn) = 1. Đặt b = (b1; b2; : : : ; bn 1). Ta có (b; bn) = 1. Theo Định lí 1.1.1 tồn tại số nguyên ln và k sao cho: bnln + bk = c1: (1.7) Kí hiệu b 0 =bi với i = 1; 2; : : : ; n 1. Ta có (b 0 ; b 0 ; : : : s; b 0 1 )=1: b i 1 2 n Theo giả thiết quy nạp tồn tại các số nguyên l1; l2; : : : ; ln 1 sao cho b1 0 l1 + b2 0 l2 + : : : + bn 0 1 l n 1 = k hay b1l1 + b2l2 + : : : + bn 1 l n 1 = bk: (1.8) Từ (1.7) và (1.8) ta suy ra b1l1 + b2l2 + : : : + bn 1ln 1 + bnln = c1 tức là (l1; l2; : : : ; ln) là nghiệm của phương trình (1.6). Định lí được chứng minh.
  • 17. Viết thuê đề tài giá rẻ trọn gói - KB Zalo/Tele : 0973.287.149 Luanvanmaster.com – Cần Kham Thảo - Kết bạn Zalo/Tele : 0973.287.149 17 Chúng ta không đi sâu vào việc tìm biểu thức cho nghiệm tổng quát của nó như đã làm đối với trường hợp n = 2. Tuy nhiên có thể thấy rằng nếu phương trình (1.5) có nghiệm nguyên a1; a2; : : : ; an thì nó sẽ có vô số nghiệm nguyên phụ thuộc vào n 1 tham số. Thật vậy, dễ dàng kiểm tra được tất cả các bộ n số nguyên x1; x2; : : : ; xn xác định như sau là nghiệm của (1.5), 8 x1 = a1 + ant1 >x2 + a2 + ant2 > > > > > > : : : > > > > < > xn 1 = an 1 + antn 1 > > > x= a n at 1 a t : : : a n 1 t n 1 > n 1 2 2 > > > > > , > được chọn tuỳ ý. t 2 Z i = 1 ; 2; : : : n 1 trong đó i : Bây giờ, ta sẽ thảo luận về cách giải phương trình (1.5). Về mặt thực hành ta có thể tiến hành theo hai cách sau đây. Cách 1. Đưa (1.5) về trường hợp có một hệ số bằng 1. Cách 2. Nếu phương trình (1.5) có hai hệ số nguyên tố cùng nhau, chẳng hạn (a1; a2) = 1 thì ta viết dưới dạng a1x1 + a2x2 = c a3x3 : : : anxn rồi giải phương trình theo hai ẩn x1, x2. Ta xét hai ví dụ sau đây, nó sẽ lần lượt minh họa cho hai cách trên. Ví dụ 1.2.2. Tìm tất cả các nghiệm nguyên của phương trình 6x + 45y + 6z 10t = 13:
  • 18. Viết thuê đề tài giá rẻ trọn gói - KB Zalo/Tele : 0973.287.149 Luanvanmaster.com – Cần Kham Thảo - Kết bạn Zalo/Tele : 0973.287.149 Lời giải. Phương trình đã cho được viết dưới dạng 6(x + z) + 10(4y t) + 5y = 13:
  • 19. Viết thuê đề tài giá rẻ trọn gói - KB Zalo/Tele : 0973.287.149 Luanvanmaster.com – Cần Kham Thảo - Kết bạn Zalo/Tele : 0973.287.149 18 Đặt x + z = x1; 4y t = x2 ta được 6x1 + 10x2 + 5y = 13. Suy ra x1 + 10x2 + 5(y + x1) = 13: Đặt x1 + y = x3 ta được x1 + 10x2 + 5x3 = 13: Vậy x1 = 13 10x2 5x3: Từ đây, bằng tính toán trực tiếp ta có y = x3 x1 = x3 (13 10x2 5x3) = 10x2 + 6x3 13; t = 4y x2 = 39x2 + 24x3 52; x = x1 z = 13 10x2 5x3 z: Vậy nghiệm tổng quát của phương trình đã cho là x = 13 10x2 5x3 x4; y = 10x2 + 6x3 13; z = x4; t = 39x2 + 24x3 52: trong đó x2, x3, x4 là các số nguyên tuỳ ý. Ví dụ 1.2.3. Giải phương trình 6x + 15y + 10z = 3: (1.9) Lời giải. Ta có thể viết (1.9) dưới dạng 6(x + z) + 15y = 3 4z: Đặt u = x + z ta có 15y + 4z = 3 6u. Ta thấy ( 1; 4) là nghiệm riêng của 15y + 4z = 1. Do đó ( 3 + 6u; 12 24u) là nghiệm riêng của 15y + 4z = 3 6u. Suy ra nghiệm tổng quát của nó là y = 3 + 6u + 4t; z = 12 24u 15t:
  • 20. Viết thuê đề tài giá rẻ trọn gói - KB Zalo/Tele : 0973.287.149 Luanvanmaster.com – Cần Kham Thảo - Kết bạn Zalo/Tele : 0973.287.149 19 Từ u = x + z suy ra x = u z = u (12 24u 15t) = 12 + 25u + 15t: Vậy nghiệm tổng quát của (1.9) là 8 >x = 12 + 25u + 15t; > > > < y = 3 + 6u + 4t; > > > >z = 12 24u 15; : với u; t 2 Z. Một số bài toán chọn lọc Bài toán 1.2.4 (Đề thi Vô định Toán Quốc tế 1983). Cho a, b, c là các số nguyên đôi một nguyên tố cùng nhau. Chứng minh rằng 2abc ab bc ca là số nguyên lớn nhất không viết được dưới dạng xbc + yca + zab với x, y, z là những số không âm. Lời giải. Bài toán tương đương với việc chứng minh rằng Khẳng định 1. phương trình xbc + yca + zab = 2abcab bc ca không có nghiệm nguyên không âm; Khẳng định 2. Nếu n > 2b ab bc ca thì phương trình có nghiệm nguyên không âm. Chứng minh Khẳng định 1. Giả sử tồn tại x0; y0; z0 2 N sao cho x0bc + y0ca + z0ab = 2abc ab bc ca: Điều này tương đương với bc(x0 + 1) + ca(y0 + 1) + ab(z0 + 1) = 2abc:
  • 21. Viết thuê đề tài giá rẻ trọn gói - KB Zalo/Tele : 0973.287.149 Luanvanmaster.com – Cần Kham Thảo - Kết bạn Zalo/Tele : 0973.287.149 20 Suy ra ab(z0 + 1) .. . c. Vì (ab; c) = 1 nên z0 + 1 .. . c. Mặt khác z0 + 1 2 N nên z0 + 1 c: Tương tự y0 + 1 b, x0 + 1 a. Vậy bc(x0 + 1) + ca(y0 + 1) + ab(z0 + 1) 3abc: Điều này vô lí. Chứng minh Khẳng định 2. Xét ab có dạng kbc + lac, với 0k a 1, 0 l b 1:. Các số này khi chia cho ab sẽ cho ta các số dư khác nhau. Thật vậy, giả sử k1bc + l1ac k2bc + l2ac (mod ab): . . Vì (ab; c) = 1 nên suy ra b(k1 k2) a(l1 . k2) . l2) . ab: Từ đó b(k1 . a và a(l1 . . . . k 2 . l 2 . k2j < a, jl1 l2j < b l2) . b. Vì (a; b) = 1 nên k1 . a và l1 . b. Mặt khác jk1 nên l1 l2 = 0, k1 k2 = 0. Do đó tồn tại x0; y0 2 Z + , 0 x0 a 1, 0y0 b 1 để cho bcx0 + acy0 n (mod ab): Vậy tồn tại z0 2 Z để bcx0 + acy0 + abz0 = n: Ta có ab(z0 + 1) = n bcx0 acy0 + ab > 2abc bcx0 acy0 = bc(a 1 x0) + ac(b Do đó z0 + 1 > 0. Suy ra z0 0. bc ca 1 y0) 0: Bài toán 1.2.5. Cho các số nguyên không âm a, b thoả mãn điều kiện 5a 7b: Chứng minh rằng hệ 8 <x + 2y + 3z + 7u = a; :y + 2z + 5u = b luôn có nghiệm nguyên không âm.
  • 22. Viết thuê đề tài giá rẻ trọn gói - KB Zalo/Tele : 0973.287.149 Luanvanmaster.com – Cần Kham Thảo - Kết bạn Zalo/Tele : 0973.287.149 21 Chứng minh. Giả sử b = 5u + v với 0 u, 0 v 4. Từ điều kiện a 7 5 b suy ra a 7(5u+v) . Từ đây ta có 5 a 7u 7v : 5 Như vậy hệ phương trình có thể viết là n x + 2y + 3z = a yu y + 2z = b 5u = v: Với mỗi 0 v 4 ta sẽ chọn y, z thích hợp để có x = z 7u 2y 3z 0. (1.10) (1.11) Nếu v = 0, ta lấy y = z = 0. Khi đó x = a 7u 7 5 v = 0 (theo (1.10))). Nếu v = 1, ta thấy y = 1, z = 0. Khi đó x = a yu 2 7v 2 = 7 2> 1: 5 5 Vậy x 0: Nếu v = 2, ta lấy y = 0, z = 1. Khi đó x = a 7u 3 7v 3 = 14 3> 1: 5 5 Vậy x 0: Nếu v = 3, ta lấy y = z = 1. Khi đó x = a 7u 5 7v 5 = 21 5> 1: 5 5 Vậy x 0: Nếu v = 4, ta lấy y = 0, z = 2. Khi đó x = a 7u 6 7v 6 = 28 6> 1: 5 5 Vậy x 0: Tóm lại với mọi v = 0; 1; 2; 3; 4 ta đều chọn được y, z, x không âm để (1.11) được nghiệm đúng. Bài toán được giải xong.
  • 23. Viết thuê đề tài giá rẻ trọn gói - KB Zalo/Tele : 0973.287.149 Luanvanmaster.com – Cần Kham Thảo - Kết bạn Zalo/Tele : 0973.287.149 22 Bài toán 1.2.6 (Đề thi vô địch Mỹ 1982). Chứng minh rằng tồn tại số tự nhiên k sao cho tất cả các số k n + 1 với n = 1; 2; : : : đều là hợp số. Lời giải. Xét các số Fermat Fm = 22m + 1: Ta đã biết Fm là số nguyên tố với m = 0; 1; 2; 3; 4, trong khi Euler phát hiệu rằng F5 là một hợp số, F5 là tích của hai số nguyên tố 641 và 6700417. Ký hiệu ai = Fi với i = 0; 1; 2; 3; 4; a5 = 641a6 = 6700417: Theo Định lí Trung Hoa về thặng dư, tồn tại số tự nhiên k > max(a0; : : : ; a6) để k 1 (mod am) với m = 0; 1; 2; : : : ; 5 và k 1 (mod a6). Ta sẽ chứng minh k chính là số cần tìm. Xét n bất kỳ, n có thể viết dưới dạng n = 2 m p với 0 m và p lẻ. (a) Nếu m 4, ta có k 2 n + 1 2n + 1 (mod am). Mặt khác 2n + 1 = 2 2m p + 1 = (2 2m )p + 1 .. . 2 2m + 1 = a m : Do đó k 2 n + 1 .. . a m . Vì k 2 n + 1 > k > a m nên k 2 n + 1 là hợp số. (b) Nếu m = 5 thì k 2 n + 1 2n + 1 (mod a5). Mặt khác 2n +1 = 225 p +1 .. . 225 +1 = F5 .. . 641 = a5. Vậy k 2n +1 .. . a5. Mặt khác k 2n + 1 > a5, suy ra k 2n + 1 là hợp số. (c) Nếu m 6, khi đó n = 26 b với b 2 Z. Ta có k 2 n + 1 2 n + 1 (mod a6); 2n = 22 5 2b = (F5 1)2b ( 1)2b 1 (mod a6) vì F5 . .. a6: Do đó k 2 n + 1 1 + 1 = 0 (mod a6). Vì k 2 n + 1 > a6 nên k 2 n + 1 là hợp số. Vậy với mọi n, số k 2n + 1 luôn là hợp số.
  • 24. Viết thuê đề tài giá rẻ trọn gói - KB Zalo/Tele : 0973.287.149 Luanvanmaster.com – Cần Kham Thảo - Kết bạn Zalo/Tele : 0973.287.149 23 Chương 2 Một số phương trình Diophantine phi tuyến 2.1 Phương trình Pell loại 1 Phương trình Pell loại 1 là phương trình có dạng x 2 Dy 2 = 1; (2.1) trong đó D 2 N và ta yêu cầu tìm nghiệm x; y 2 N . Trong tiết này khi nói đến nghiệm của (2.1) ta hiểu là nghiệm nguyên dương. Định lí 2.1.1 (Điều kiện tồn tại nghiệm). Phương trình (2.1) có nghiệm nguyên dương khi và chỉ khi D là số không chính phương. Chứng minh. Giả sử D = m 2 . Khi đó x 2 Dy 2 = x 2 m 2 y 2 = 1 ! (x my)(x + my) = 1 ! x my = x + my = 1 ! x = 1; y = 0: Vậy (2.1) không có nghiệm nguyên dương. Ngược lại giả sử D là số không chính phương. Ta có các bổ đề sau Bổ đề 2.1.2. Cho a 2= Q. Khi đó tồn tại vô số cặp nguyên (h; k) với k > 0 sao cho a k < k 1 2 : h
  • 25. Viết thuê đề tài giá rẻ trọn gói - KB Zalo/Tele : 0973.287.149 Luanvanmaster.com – Cần Kham Thảo - Kết bạn Zalo/Tele : 0973.287.149 24 Chứng minh. Ta sử dụng nhận xét đã biết sau: Với mỗi q 2 N tồn tại cặp số nguyên (h; k) với 1 k q sao cho a Ký hiệu h < 1 : k kq A = (h; k) : a k < k 1 2 : h Ta phải chứng minh j A j j j < ¥. Khi đó tồn tại e sao cho = ¥. Giả sử trái lại A a k > e với mọi (h; k) 2 A. Chọn q 2 N sao cho h 1 < e: (2.2) q Theo nhận xét trên tồn tại cặp số nguyên (h0; k0) với 1 q sao cho a k0 < k0q k0 : (2.3) h0 1 12 Từ (2.3) ta có (h0; k0) 2 A ! h0 1 1 a h0 > e. Mâu a k0 > e. Nhưng q k0q > k0 thuẫn với (2.2). Bổ đề 2.1.3. Tồn tại vô số cặp số nguyên dương (x; y) sao cho jx 2 Dy 2 j < 1 + 2 p : D Chứng minh. Theo Bổ đề 2.1.2 tồn tại vô số cặp số nguyên (x; y) sao cho 0 < pD y < y 1 2 : x Suy ra y + pD = y p D + 2p D < y 1 2 + 2p D: x x Vậy y y + p D jx2 Dy2j =jx ypDjjx + ypDj = y2 pD x x 2 1 1 p 1 p p < y ( +2 D)= + 2 D 1 + 2 D y2 y2 y2 < :
  • 26. Viết thuê đề tài giá rẻ trọn gói - KB Zalo/Tele : 0973.287.149 Luanvanmaster.com – Cần Kham Thảo - Kết bạn Zalo/Tele : 0973.287.149 25 Chứng minh định lý Từ bổ đề 2.1.3 (x; y) tồn tại vô số cặp số nguyên dương (x; y) sao cho jx 2 Dy 2 j < 1 + 2 p : D p p Đặt I = [ 1 2 d; 1 + 2 d]. Với mỗi k 2 I ký hiệu Ak = f(x; y) 2 N : x 2 Dy 2 = kg Do đó tồn tại k 2 I để jAkj = ¥. Suy ra tồn tại (x1; y1) 6= (x1; y1) 2 Ak để x1 x2 (mod jkj) y1 y2 (mod jkj); x1 2 Dy1 2 = x2 2 Dy2 2 = k; Xét tích (x1 y1 p )(x2 + y2 p )x1x2 Dy1y2 + p (x1y2 x2y1) (2.4) D D D Vì x1x2 Dy1y2 x1 2 Dy1 2 0 (mod jkj) x1y2 x2y1 x1y1 x1y1 0 (mod jkj): Vậy tồn tại u; v 2 Z sao cho x1x2 Dy1y2 = ku (2.5) x1y2 Dy1x2 = kv (2.6) Từ (2.4), (2.5), (2.6) suy ra (x1 y1 p )(x2 + y2) = k(u + v p ); D D (x1 + y1)(x2 y2 p ) = k(u v p ): D D Nhân hai đẳng thức trên với nhau và chú ý rằng (x1; y1; (x2; y2) 2 Ak ! x1 2 dy1 2 = x2 2 dy2 2 = k ta được k2 = k2(u2 Dv2) ! u2 Dv2 = 1:
  • 27. Viết thuê đề tài giá rẻ trọn gói - KB Zalo/Tele : 0973.287.149 Luanvanmaster.com – Cần Kham Thảo - Kết bạn Zalo/Tele : 0973.287.149 26 Ta chứng minh u; v > 0. Rõ ràng u > 0. Nếu trái lại v = 0 thì u = 1 ! (x1 y1 p )(x2 + y2 p ) = k = (x1 2 Dy1 2 ) = (x1 y1 p )(x1 + y1 p ) ! x2 + D D D D p p y 2 D = x1 + y1 D ! x1 = x2; y1 = y2. Ta có mâu thuẫn. Vậy (u; v) là nghiệm nguyên dương của phương trình (2.1). Định lí 2.1.4 (Công thức nghiệm). Ký hiệu (a; b) là nghiệm nhỏ nhất của phương trình x2 Dy2 = 1: Khi đó dãy (xn; yn) cho bởi (a + b p )n + (a b p ) n D D xn = ; 2 (a + b p ) n (a b p ) n D D yn = 2p : D cho tất cả các nghiệm của (2.2) Dãy nghiệm (xn; yn) cũng có thể xác định theo công thức truy hồi sau x0 = 1; x1 = a; xn+2 = 2axn+1 xn; (2.7) y0 = 0; y1 = b; yn+2 = 2ayn+1 yn: (2.8) Chứng minh. Ta có xn + yn p = (a + b p ) n ; xn yn = (a bp ) n : (2.9) D d d Suy ra (xn 2 Dy 2 n) = (a2 Db2) n = 1. Đảo lại giả sử (x; y) là nghiệm bất kỳ của (2.1). Ta chứng minh tồn tại n 2 N để x = xn; y = yn. Vì D không chính phương nên điều này tương đương Tồn tại n 2 N để x + y p D = xn + yn p D = (a + b p D) n : Chứng minh bằng phản ứng. Giả sử trái lại x + y p D 6= (a + b p D) n 8n 2 N :
  • 28. Viết thuê đề tài giá rẻ trọn gói - KB Zalo/Tele : 0973.287.149 Luanvanmaster.com – Cần Kham Thảo - Kết bạn Zalo/Tele : 0973.287.149 27 Khi đó tồn tại m 2 N sao cho (a + b p D) m < x + y p D < (a + b p D) m+1 : Nhân hai vế với (a bp ) m ta được D 1 < (x + y p )(a b p ) m < a + b p : D D D Do (2.9) ta có (x + y p )(a b p )m = (x + y p )(xm ym p ) D D D D = (xxm Dyym) + (xmy ymx) p D = u + v p ; D ở đó u = xxm Dyym; v = xmy ymx. Vậy 1 < u + v p < a + b p : (2.10) d d Ta có u2 Dv2 = (xxm Dyym) 2 D(xmy ymx)2 ; = (x 2 Dy 2 )(xm 2 Dy 2 m) = 1: p p p p Lại có x > y d; xm > ym d nên u > 0. Lại có (u v D)(u + v D) = 1 và 1 < p p p u + v d nên 0 < u v d < 1 < u + v d ! v > 0. Vậy (u; v) là nghiệm của (2.1) p p do đó a u; b u ! a + b d u + v d trái với (2.10). Định lý được chứng minh. Từ định lý trên ta thấy việc tìm nghiệm của phương trình Pell (2.1) quy về tìm nghiệm nhỏ nhất (a; b) của nó. Cách đơn giản nhất là thử bằng tay: Thay lần lượt y = 1; 2; : : : vào biểu thức 1 + Dy 2 cho tới khi nào được số chính phương thì dừng lại. Vì phương trình (2.1) có nghiệm nên chắc chắn quá trình này sẽ dừng lại sau p b phép thử. Khi đó nghiệm nhỏ nhất là (a; b) với a = 1 + Db 2 . Nếu nghiệm nhỏ
  • 29. Viết thuê đề tài giá rẻ trọn gói - KB Zalo/Tele : 0973.287.149 Luanvanmaster.com – Cần Kham Thảo - Kết bạn Zalo/Tele : 0973.287.149 28 nhất b này lớn thì cách thử này không khả thi. Thí dụ với phương trình x 2 61y 2 thì cặp nghiệm nhỏ nhất là a = 1766319049; b = 226153980: Sau đây ta sẽ trình bày một thuật toán sử dụng liên phân số để tìm một nghiệm nhỏ nhất (a; b) của (2.1). Định nghĩa 2.1.5. Cho a0; a1; a2; : : : là dãy số nguyên trong đó ai > 0; i 1. Đặt Ck = [a0; a1; : : : ; ak]: Khi đó tồn tại giới hạn limCk = a: Ta gọi a là giá trị của liên phân số vô hạn [a0; a1; a2; : : :]và viết a = [a0; a1; a2; : : :]: Định lí 2.1.6. a = [a1; a1; a2; : : :] là một số vô tỷ. Ngược lại mỗi số vô tỷ đều biểu diễn duy nhất dưới dạng một liên phân số vô hạn. Định nghĩa 2.1.7. Ta gọi liên phân số vô hạn a = [a1; a1; a2; : : :] là tuần hoàn nếu dãy (an) tuần hoàn kể từ một chỉ số nào đó tức là: Tồn tại các số nguyên dương m và k sao cho mọi n m ta có an = am+k. Số nguyên dương k được gọi là chu kỳ của liên phân số a = [a0; a1; a2; : : :]. Khi đó ta viết a = [a0; a1; a2; : : : ; am 1; am; am+1; : : : ; am+k 1]: p Nếu D là số không chính phương, biểu diễn liên phân số của D được cho bởi p Định lí 2.1.8. Biểu diễn liên phân số của D là tuần hoàn và có dạng p D = [a; a1; a2; : : : ; an; 2a]: p với a = [ D]. Hơn nữa có công thức tường minh để xác định dãy (a1; : : : ; an). Chú ý rằng dãy (a1; : : : ; an) là đối xứng tức là
  • 30. Viết thuê đề tài giá rẻ trọn gói - KB Zalo/Tele : 0973.287.149 Luanvanmaster.com – Cần Kham Thảo - Kết bạn Zalo/Tele : 0973.287.149 a1 = an; a2 = an 1; : : :
  • 31. Viết thuê đề tài giá rẻ trọn gói - KB Zalo/Tele : 0973.287.149 Luanvanmaster.com – Cần Kham Thảo - Kết bạn Zalo/Tele : 0973.287.149 29 Ví dụ 2.1.9. p 23 = [4;1;3;1;8] p 29 = [5;2;1;1;2;10] p 31 = [5;1;1;3;5;3;1;1;10] p 46 = [6;1;2;1;1;2;6;2;1;1;2;1;12] p 76 = [8;1;2;1;1;5;4;5;1;1;2;1;16] p 97 = [9;1;5;1;1;1;1;1;1;5;1;18]: Định lí 2.1.10. Cho phương trình Pell x 2 Dy 2 = 1: (I) 1. Biểu diễn p thành liên phân số D p D = [a; a1; a2; : : : ; an; 2a]: 2. Nếu chu kỳ n là số chẵn ta tính giản phân thứ n1 pn 1 Cn 1 = : 3. Khi đó (pn 1; qn 1) là nghiệm nhỏ nhất của (2.1) 4. Nếu chu kỳ n là số lẻ ta tính giản phân thứ 2n 1 C 1 = p 2n 1 : 2n q 2n 1 5. Khi đó (p2n 1; q2n 1) là nghiệm nhỏ nhất của (2.1) Ví dụ 2.1.11. Tìm nghiệm nhỏ nhất của phương trình x 2 14y 2 = 1. Ta có p = 14 ]. Chu kỳ n = 4 là số chẵn. Vậy ta tính giản phân [3;1;2;1;6 1 C3 = [3;1;2;1] = 3+ 1 1 + 1 2 + 1
  • 32. Viết thuê đề tài giá rẻ trọn gói - KB Zalo/Tele : 0973.287.149 Luanvanmaster.com – Cần Kham Thảo - Kết bạn Zalo/Tele : 0973.287.149 30 = 15: 4 Vậy nghiệm nhỏ nhất là (15; 4). p Ví dụ 2.1.12. Tìm nghiệm nhỏ nhất của phương trình x2 13y2 = 1. Ta có 13 = [3;1; 1; 1; 1; 6] = [3; 1; 1; 1; 1; 6; 1; 1; 1; 1; 6; : : : Chu kỳ n = 5 là số lẻ. Vậy ta tính giải phân C9 = [3;1;1;1;1;6;1;1;1] = 3+ 1 =649 : 1 1 + 180 1+ .. . + 1 1 1 + 1 Vậy nghiệm nhỏ nhất là (649; 180) Trở lại phương trình x 2 61y 2 = 1. Ta có p ] 76 = [7;1;4;3;1;2;2;1;3;4;1;14 Chu kỳ n = 11 là số lẻ. Ta tính giản phân C21 = [7;1;4;3;1;2;2;1;3;4;1;14;1;4;3;1;2;2;1;3;4;1] = 7 + 1 1 1 + 1 4 + 1 3+ .. . + 4 + 1 1 = 1766319049 226153980 Vậy nghiệm nhỏ nhất là (1766319049; 226153980) 2.2 Phương trình Pell loại 2 Phương trình Pell loại 2 là phương trình x 2 Dy 2 = 1 (2.11) ở đó D 2 N . Nghiệm của (2.11) luôn được hiểu là nghiệm nguyên dương.
  • 33. Viết thuê đề tài giá rẻ trọn gói - KB Zalo/Tele : 0973.287.149 Luanvanmaster.com – Cần Kham Thảo - Kết bạn Zalo/Tele : 0973.287.149 31 Định lí 2.2.1. Điều kiện cần để (2.11) có nghiệm là D không là số chính phương và D không có ước nguyên tố dạng 4k + 3. Chứng minh. i) nếu D = m 2 thì (2.11) trở thành (my x)(my +x) = 1 ! my x = my + x = 1 ! x = 0 vậy (2.11) vô nghiệm. ii) Nếu D có ước nguyên tố p = 4k +3 : Giả sử (x; y) là nghiệm. Khi đó x 2 +1 = Dy 2 ! pjx 2 + 1. Vì p = 4k + 3 nên pj1. Mâu thuẫn. Tuy nhiên, đây không là điều kiện đủ. Định lí 2.2.2. Nếu D = p là số nguyên tố thì (2.11) có nghiệm khi và chỉ khi p = 2 hoặc p 6= 4k + 3. Chứng minh. Nếu (2.11) có nghiệm thì từ Định lý 1 suy ra p = 2 hoặc p 6= 4k + 3. Đảo lại nếu p = 2 thì phương trình x2 = 2y2 = 1 có nghiệm (x; y) = (1; 1). Xét p 1( mod 4). Xét phương trình Pell loại I x2 Dy2 = 1 (2.12) Gọi (a; b) là nghiệm nhỏ nhất của (2.12). Ta có a2 1 = pb2. Nếu a chẵn thì b lẻ do đó b 2 1( mod 4) ! a 2 1 + p 2( mod 4). Mâu thuẫn. Vậy a lẻ, b chẵn. Đặt a = 2a1 + 1; b = 2b1. Ta có (a 1)(a + 1) = pb2 , a1(a1 + 1) = pb2 1. Do p nguyên tố và (a1; a1 +1) = 1 nên a1 = u 2 ; a1 +1 = pv 2 hoặc a1 = pu 2 ; a1 +1 = v 2 . Nếu a1 = u 2 ; a1 + 1 = pv 2 ! u 2 pv 2 = 1. Vậy (2.11) có nghiệm (u; v). Nếu a1 = pu 2 ; a1 + 1 = v 2 ! v 2 pu 2 = 1. Vậy (v; u) là nghiệm của (2.12). Suy ra u a ! a1 + 1 = v 2 v a = 2a1 + 1. Mâu thuẫn. Định lí 2.2.3. Gọi (a; b) là nghiệm nhỏ nhất của (2.12). Khi đó (2.11) có nghiệm khi và chỉ khi 8 > a = x2 + Dy2 < (2.13) > b = 2xy :
  • 34. Viết thuê đề tài giá rẻ trọn gói - KB Zalo/Tele : 0973.287.149 Luanvanmaster.com – Cần Kham Thảo - Kết bạn Zalo/Tele : 0973.287.149 32 có nghiệm nguyên dương. Hơn nữa nếu (2.13) hệ có nghiệm nó sẽ có nghiệm duy nhất. Nghiệm duy nhất (x1; y1) này chính là nghiệm nhỏ nhất của (2.11). Chứng minh. 1) Giả sử (2.11) có nghiệm. Gọi (x1; y1) là nghiệm nhỏ nhất của (2.11). Đặt u = x1 2 +Dy2 1; v = 2x1; y1. Ta chứng minh u = a; v = b do đó (x1; y1) chính là nghiệm của (2.13). Ta có u2 Dv2 = (x1 2 Dy2 1)2 = 1. Vậy (u; v) là nghiệm của (2.12). Suy ra u a; v b. Ta có (a 2 Db 2 )(x1 Dy1 2 ) = 1( 1) = 1 ,(ax1 Dby1) 2 D(ay1 bx1) 2 = 1: Dễ thấy (ax1 Dby1)2 > 0; (ay1 bx1)2 > 0. Do (x1; y1) là nghiệm của (2.11) nên (ax1 Dby1) 2 x1 2 , a 2 x1 2 + D 2 b 2 y1 2 x1 2 2abDx1y1 ! x1 2 (a2 1) + D2 b2 y1 2 2abDx1y1 ! x1 2 Db 2 + D 2 b 2 y 2 1 2abDx1y1 ! b(x1 2 + dy 2 1) 2ax1y1 ! bu av ! b 2 u 2 a 2 v 2 ! b 2 (Dv 2 + 1) v 2 (Db 2 + 1) ! b v ! b = v ! u = a: 2) Đảo lại giả sử (u; v) là nghiệm của (2.13). Ta có a2 Db2 = 1 , (u2 +Dv2 )2 D(2uv) 2 = (u 2 Dv 2 ) 2 = 1. Vậy u 2 Dv 2 = 1 hoặc u 2 Dv 2 = 1. Nếu u 2 Dv 2 = 1 thì (u; v) là nghiệm của (2.12) do đó u a = u 2 + Dv 2 . Mâu thuẫn. Vậy u2 Dv2 = 1 do đó (2.11) có nghiệm (u; v). Tiếp theo ta chứng minh (u; v) là nghiệm nhỏ nhất của (2.11). Giả sử (x1; y1) là nghiệm nhỏ nhất của (2.11). Theo chứng minh ở 1) ta có a = u 2 + Dv 2 = x1 2 + p Dy 2 1; b = 2uv = 2x1y1 ! u 2 + Dv 2 + 2uv = x1 2 + Dy 2 1 + 2x1y1 ! (u + v D) 2 = (x1 + y1) 2 ! u = x1; v = y1. Định lý được chứng minh.
  • 35. Viết thuê đề tài giá rẻ trọn gói - KB Zalo/Tele : 0973.287.149 Luanvanmaster.com – Cần Kham Thảo - Kết bạn Zalo/Tele : 0973.287.149 33 Ví dụ là một áp dụng của định lý 2. Nó cũng chỉ ra rằng điều kiện của định lý 1 chỉ là điều kiện cần. Ví dụ 2.2.4. Chứng minh rằng phương trình x 2 34y 2 = 1 vô nghiệm (Ở đây 34 = 2:17 không số chính phương và cũng không có ước nguyên tố dạng 4k + 3). Giải Phương trình x 2 34y 2 = 1 có nghiệm nhỏ nhất là (a; b) = (35; 6). Xét hệ 8 <35 = x 2 + 34y 2 :6 = 2xy Từ phương trình thứ nhất của hệ suy ra (x; y) = (1; 1). Tuy nhiên (1; 1) không thoả mãn phương trình thứ hai. Vậy hệ vô nghiệm do đó theo định lý 2 phương trình x 2 34y 2 = 1 vô nghiệm. Ta thừa nhận định lý Định lí 2.2.5. Phương trình Pell loại 2 x 2 Dy 2 = 1 p có nghiệm khi và chỉ khi trong biểu diễn D thành liên phân số p D = [a; a1; a2; : : : ; an; 2a] chu kỳ n là số lẻ. Trong trường hợp đó (pn 1; qn 1) là nghiệm nhỏ nhất của phương trình ở đó. pn 1 Cn 1 = là giải phân thứ n 1. p Ví dụ 2.2.6. i) Xét phương trình x 2 13y 2 = 1. Ta có D = [3;1; 1; 1; 1; 6]. Chu kỳ n = 5 là số lẻ. Vậy phương trình có nghiệm. Ta tính giản phân
  • 36. Viết thuê đề tài giá rẻ trọn gói - KB Zalo/Tele : 0973.287.149 Luanvanmaster.com – Cần Kham Thảo - Kết bạn Zalo/Tele : 0973.287.149 C4 = [3;1;1;1;1]
  • 37. Viết thuê đề tài giá rẻ trọn gói - KB Zalo/Tele : 0973.287.149 Luanvanmaster.com – Cần Kham Thảo - Kết bạn Zalo/Tele : 0973.287.149 34 = 3 + 1 1 1 + 1 1 + 1 1 + 3 18 1 = 3 + = 5 5 Vậy nghiệm nhỏ nhất là (18; 5) ii) Xét phương trình x 2 34y 2 = 1. Ta có p ]. Chu kỳ n = 4 34 = [5;1;4;1;10 là số chẵn. Vậy phương trình vô nghiệm. Định lí 2.2.7 (Công thức nghiệm). Giả sử phương trình Pell loại 2 x 2 Dy 2 = 1 (II) có nghiệm. Gọi (a; b ) là nghiệm nhỏ nhất của nó. Khi đó dãy (xn; yn)¥ cho bởi n=1 (a + b p )2n 1 + (a b p )2n 1 xn = D D (a + b p 2 p yn = D)2n 1 + (a b D)2n 1 2pD cho ta tất cả các nghiệm của (II). Chứng minh. Giả sử (xn; yn) cho bởi công thức trên. Khi đó xn + yn p D = (a + b p D) 2n 1 xn yn p = (a b p )2n 1 D D ! xn 2 Dyn 2 = (a2 Db 2 ) 2n 1 = 1 Ngược lại giả sử (x; y) là một nghiệm của (2.11). Ta có p p p p = s +t D ởđó s = xa + Dyb ;t = ya + xb ! s2 Dt2 = (xa + Dyb )2 D(ya + xb ) 2
  • 38. Viết thuê đề tài giá rẻ trọn gói - KB Zalo/Tele : 0973.287.149 Luanvanmaster.com – Cần Kham Thảo - Kết bạn Zalo/Tele : 0973.287.149 35 = (x 2 Dy 2 )(a 2 Db 2 ) = ( 1)( 1) = 1 Vậy (s;t) là nghiệm của phương trình Pell loại 1 s 2 Dt 2 = 1. Gọi (a; b) là nghiệm nhỏ nhất của nó. Theo công thức nghiệm của phương trình Pell loại 1 và định lý 3 tồn tại n 2 N sao cho (x + y p D)(a + b p D) = s +t p D = (a + b p D) n = (a 2 + Db 2 + 2ab p D) n = (a + b p D) 2 ) n = (a + b p D) 2n ! x + y p D = (a + b p D)2n 1 = xn + yn p D ! x = xn; y = yn Một số bài toán chọn lọc Bài toán 2.2.8. Số nguyên dương S được gọi là số Heron nếu nó là diện tích của một tam giác có ba cạnh là ba số nguyên liên tiếp. Chứng minh rằng S là số Heron khi và chỉ khi S khi là số hạng của dãy (Sn); n 1 xác định bởi S0 = 0; S1 = 6; S2 = 84; Sn+2 = 14Sn+1 Sn Lời giải. Gọi S là diện tích của tam giác có ba cạnh là x 1; x; x + 1 với x > 2; x 2 N . Theo công thức Heron. 1 x q 3(x 2 4) ! 16S 2 = 3x 2 (x 2 (2.14) S = 1) 4 Vậy S là số Heron khi và chỉ khi phương trình (2.14) có nghiệm nguyên dương(S; x). Dễ thấy x phải chẵn. Đặt x = 2y ta có 16S 2 = 3x 2 (x 2 1) , S 2 = 3y 2 (y 2 1) ! S = y q 3(y 2 1) ! 3(y 2 = 1) = h 2
  • 39. Viết thuê đề tài giá rẻ trọn gói - KB Zalo/Tele : 0973.287.149 Luanvanmaster.com – Cần Kham Thảo - Kết bạn Zalo/Tele : 0973.287.149 36 ! h = 3z ! 3(y 2 1) = 9z 2 ! y 2 3z 2 = 1; S = 3yz Ngược lại nếu (y; z) là nghiệm của phương trình Pell y2 3z2 = 1 (2.15) thì x = 2y; y > 1; S = 3yz là nghiệm của (2.14) Nghiệm nhỏ nhất của (2.15) là (2; 1) Vậy tất cả nghiệm của (2.15) (yn; zn) cho bởi (2 + p ) n + (2 p )n yn = 3 3 (2 + p ) n 2 p ) n zn = 3 p (2 3 2 3 Do đó p p )n (7 4 p Sn = 3ynzn = 3 ((7 + ) n ) 3 3 4 ! Sn+2 = 14Sn+1 Sn; S0 = 0; S1 = 6; S2 = 84 Bài toán 2.2.9. Tìm tất cả các số nguyên dương n sao cho 2n + 1 và 3n + 1 đều là số chính phương. Lời giải. Vì (2n + 1; 3n + 1) = 1 nên 2n + 1 và 3n + 1 đều là số chính phương khi và chỉ khi (2n + 1)(3n + 1) = y2 ; y 2 N . Suy ra (12n + 5)2 24y2 = 1. Đặt x = 12n + 5 ta có x2 24y2 = 1. Gọi (xk; yk) là nghiệm của nó. Nghiệm nhỏ nhất của phương trình Pell này là (5; 1). Do đó (xk) cho bởi hệ thức. x0 = 1; x1 = 5; xk+2 = 10xk+1 xk: Dễ chứng minh xk 5 (mod 12) khi vào chỉ khi k lẻ. Vậy n = nk; k 1 ở đó nk = x 2k+1 5 12 Ta xác định hệ thức truy hồi của dãy (nk)
  • 40. Viết thuê đề tài giá rẻ trọn gói - KB Zalo/Tele : 0973.287.149 Luanvanmaster.com – Cần Kham Thảo - Kết bạn Zalo/Tele : 0973.287.149 37 Đặt uk = x2k+1 = 12nk + 5. Ta có x 2k+3 = 10x 2k+2 x 2k+1 = 10(10x 2k+1 x 2k ) x 2k+1 = 100x 2k+1 10x 2k x 2k+1 = 99x 2k+1 = x 2k+1 x 2k 1 = 98x 2k+1 x 2k 1 ! uk+2 = 98uk+1 uk , 12nk+2 + 5 = 98(12nk+1 + 5) = 12nk 5 nk+2 = 98nk+1 nk + 40 với n1 = 40; n2 = 3960 Bài toán 2.2.10. Cho dãy (xn; yn) xác định như sau (x0; y0) = (0; 1); (x1; y1) = (3; 5) và 8 > xn+1 = 3xn + 2yn + 1 < (2.16) > y = 4x + 3y + 2 : n+1 n n Chứng minh rằng (xn; yn) là tất cả các nghiệm nguyên dương của phương trình x2 + (x + 1)2 = y2 Lời giải. Phương trình đã cho tương đương với (2x + 1)2 2y2 = 1 (2.17) Đặt u = 2x +1; ! u 2 2y 2 = 1. Nghiệm nhỏ nhất của phương trình này là (1:1). Do vậy dãy nghiệm (un; yn) cho bởi (1 + p ) 2n+1 + (1 p )2n+1 un = 2 2 (1 + p )2n+1 2 p )2n+1 yn = 2 p (1 2 2 2 Từ đó u0 = 1; u1 = 7; uk+2 = 6uk+1 uk
  • 41. Viết thuê đề tài giá rẻ trọn gói - KB Zalo/Tele : 0973.287.149 Luanvanmaster.com – Cần Kham Thảo - Kết bạn Zalo/Tele : 0973.287.149 38 y0 = 1; y1 = 5; yk+2 = 6yk+1 yk Ta có un = 2xk+2 +1 ! 2xk+2 +1 = 6(2nk+1 +1) 2nk 1 ! x0 = 0; x1 = 3; xk+2 = 6xk+1 xk + 2. Thành thử dãy nghiệm (xn; yn) của (2.17) cho bởi x0 = 0; x1 = 3; xn+2 = 6xn+1 xn + 2 y0 = 1; y1 = 5; yn+2 = 6yn+1 = yk Thành thử ta chỉ cần chứng minh dãy (2.16) thoả mãn quan hệ x n+2 = 6x n+1 x n + 2 y n+2 = 6y n+1 y k Thật vậy đặt zn = 2xn + 1. Khi đó dễ kiểm tra zn+1 = 3zn + 4yn yn+1 = 2zn + 3yn ! zn+2 = 3zn + 1 + 4yn+1 = 3zn+1 + 8zn + 12yn = 3zn+1 + 8zn + 3zn+1 9zn = 6zn+1 zn ! 2xn+2 + 1 = 6(2xn+1 + 1) 2xn 1 ! xn+2 = 6xn+1 xn + 2 Tương tự yn+2 = 6yn+1 yk 2.3 Phương trình Pythagoras Trong mục này chúng ta sẽ đi tìm tất cả các nghiệm nguyên dương của phương trình x 2 + y 2 = z 2 (2.18) và xét một số ứng dụng của nó. Những phương trình Diophantine này được gọi là các phương trình Pythagoras.
  • 42. Viết thuê đề tài giá rẻ trọn gói - KB Zalo/Tele : 0973.287.149 Luanvanmaster.com – Cần Kham Thảo - Kết bạn Zalo/Tele : 0973.287.149 39 Định nghĩa 2.3.1. Một bộ ba số nguyên dương (x; y; z) thoả mãn phương trình (2.18) gọi là một bộ ba số Pythagoras. Các bộ ba số Pythagora biểu thị độ dài các cạnh của một tam giác vuông. Từ một bộ ba số Pythagoras (x; y; z) nào đó ta suy ra dễ dàng một tập hợp vô hạn các bộ ba số Pythagoras khác (tx;ty;tz) với t là số nguyên dương. Ngược lại, cho (x; y; z) là một bộ ba số Pythagoras bất kỳ và giả sử d = (x; y; z). Khi đó (x1; y1; z1) cũng là một bộ ba số Pythagoras, ở đó x 1 =x ; y 1 = y ; z 1 =z ; và (x1; y1; z1) = 1: d d d Định nghĩa 2.3.2. Một bộ ba số Pythagoras (x; y; z) là nguyên thuỷ nếu (x; y; z) = 1: Rõ ràng từ lý luận trên ta chỉ cần đi tìm các bộ ba số Pythagoras nguyên thuỷ. Dễ dàng nhận thấy nếu (x; y; z) là một bộ ba số Pythagoras nguyên thuỷ thì chúng đôi một nguyên tố cùng nhau. Thật vậy, giả sử d = (x; y). Khi đó d 2 là ước của z 2 nên d là ước của z do đó d là ước chung của x, y, z. Vậy d = 1. Định lí sau đây cho công thức mô tả tất cả các bộ ba số Pythagoras nguyên thuỷ. Định lí 2.3.3. Giả sử (x; y; z) là một bộ ba số Pythagoras nguyên thuỷ. Khi đó x và y có tính chẵn lẻ khác nhau. Nếu x chẵn, y lẻ chẳng hạn (x; y; z) có dạng x = 2mn; y = m 2 n 2 ; n = m 2 + n 2 trong đó m, n là hai số nguyên đương nguyên tố cùng nhau, có tính chẵn lẻ khác nhau. Đảo lại, nếu m, n là hai số nguyên dương nguyên tố cùng nhau, một chẵn, một lẻ thì ba bộ (x; y; z) xác định như trên là một bộ ba số Pythagoras nguyên thuỷ. Chứng minh. Trước hết vì (x; y) = 1 nên x và y không cùng chẵn. Hai số x và y cũng không thể cùng lẻ vì nếu như vậy thì z2 = x2 + y2 2 (mod 4), điều này không thể có vì một số chính phương chỉ đồng dư 0 hoặc 1 theo modulo 4.
  • 43. Viết thuê đề tài giá rẻ trọn gói - KB Zalo/Tele : 0973.287.149 Luanvanmaster.com – Cần Kham Thảo - Kết bạn Zalo/Tele : 0973.287.149 40 Giả sử x chẵn, y lẻ. Khi đó z lẻ và x 2 = z 2 y 2 (z + y)(z y), điều này tương đương với 2 2 = 2 2 : (2.19) x z + y z y z+y z y z+y = m 2 và Dễ thấy rằng ; = 1 vì (z; y) = 1. Suy ra tồn tại m; n 2 N để 2 2 2 z y = n 2 . Từ đó x = 2mn, y = m 2 n 2 , z = m 2 + n 2 . Mặt khác (m 2 ; n 2 ) = 1 nên 2 (m; n) = 1. Hơn nữa vì y và z lẻ nên m, n có tính chẵn lẻ khác nhau. Bây giờ ta chứng minh phần đảo của định trong định lí trên. Dễ dàng kiểm tra rằng x, y, z xác định theo công thức trên là bộ ba số Pythagoras. Ta còn phải chứng minh đó là bộ ba số Pythagoras nguyên thuỷ. Giả sử d = (y; z): Khi đó vì y, z lẻ nên d lẻ. Ta có d j y + z = 2m2 suy ra d j m2 và d j z y = 2n2 suy ra d j n2 : Vì (m; n) = 1 nên từ đó d = 1. Vậy (y; z) = 1 suy ra (x; y; z) = 1. Định lí được chứng minh. Sau đây là áp dụng quan trọng của Định lí 2.3.3. Định lí 2.3.4. Không tồn tại hai số tự nhiên x và y để tổng các bình phương và hiệu các bình phương của chúng đều là các số chính phương. Chứng minh. Ta chứng minh bằng phản chứng. Giả sử tồn tại các số nguyên dương x, y, có tính chất đã nêu. Trong các cặp số (x; y) ta chọn (x0; y0) là cặp số mà x nhỏ nhất. Giả sử 8 x0 2 + y0 2 = z0 2 ; 2 2 2 (2.20) < x0 y0 = t0 ; : với z0; t0 N . Ta có (x0; y0) = 1. Thật vậy, giả sử d = (x0; y0). Từ (2.20) suy ra d2 d2 j t0 2 . Từ đó d j z0 , d j t0 . Đặt x0 y1 y0 z1 = y z0 t1 t0 ; = ; ; = : d d d d j z 2 0 và
  • 44. Viết thuê đề tài giá rẻ trọn gói - KB Zalo/Tele : 0973.287.149 Luanvanmaster.com – Cần Kham Thảo - Kết bạn Zalo/Tele : 0973.287.149 41 Ta có x1 2 + y 2 1 = z 2 1; x1 2 y 2 1 = t1 2 : Suy ra (x1; y1) là một cặp thoả mãn tính chất đã nêu. Do đó x1 2 + y2 1 x0 2 + y2 0 = d(x1 2 + y2 1): Vậy d = 1: Từ (2.20) suy ra z2 0 +t0 2 = 2x0 2 . Do đó z0 và t0 có cùng tính chẵn lẻ. Đặt z0 +t0 z0 t0 u = ; v = 2 2 với u, v là các số nguyên dương. Khi đó u + v = z0; u v = t0vu2 + v2 = x0 2 : (2.21) Ta có (u; v) = 1, vì nếu d = (u; v) thì d 2 j x0 2 . Mặt khác d j u + v = z0, suy ra d 2 j z 2 0 x0 2 = y 2 0 suy ra d j y0. Vậy d = 1: Theo (2.21) thì (u; v; x0) là một bộ ba số Pythagoras nguyên thuỷ. Theo Định lí 2.3.3 tồn tại các số nguyên dương m, n chẵn lẻ khác nhau với (m; n) = 1, m > n sao cho hoặc u = 2mn; u = 2m2 v = m2 n2; n2; v = 2mn: Trong mọi trường hợp, uv = 2mn(m2 n2). Ta lại có 2y = (u + v)2 (u v)2 = 4uv = 8mn(m2 n2) suy ra y = 4mn(m2 n2), tức là y0 = 2k. Vậy mn(m2 n2) = k2: (2.22) Ta có (m; n) = 1 nên (m + n; m) = 1 và (m n; m) = 1. Vậy (m2 n2; m) = 1:
  • 45. Viết thuê đề tài giá rẻ trọn gói - KB Zalo/Tele : 0973.287.149 Luanvanmaster.com – Cần Kham Thảo - Kết bạn Zalo/Tele : 0973.287.149 42 Tương tự, (m2 n2 ; n) = 1. Do điều này nên từ (2.3) suy ra tồn tại a; b; c 2 N để m = a2; n = b2; m 2 n2 = c2: (2.23) Gọi d = (m + n; m n). Do m, n chẵn lẻ khác nhau nên m + n, m n lẻ, từ đó d lẻ. Vì d j m + n, d j m n nên d j 2m, d j 2n suy ra d j m, d j n (do d lẻ). Suy ra d = 1. Do điều này nên từ (2.23) suy ra tồn tại r; s 2 N để m n = r 2 ; m + n = s 2 : Vậy a2 + b2 = s 2 ; a 2 b2 = r 2 ; tức là cặp (a; b) thoả mãn tính chất đã nêu trong định lí. Mặt khác z0 +t0 a 2 + b 2 = m + n 2m 2mn u = < z0 z0 2 = x0 2 = y0 2 : 2 Điều này trái với cách chọn cặp (x0; y0). Định lí được chứng minh xong. Một số bài toán chọn lọc Bài toán 2.3.5 (Đề dự tuyển thi Toán quốc tế năm 1979). Chứng minh rằng không tồn tại hình chóp tứ giác đều mà các cạnh, diện tích toàn phần và thể tích của nó đều là các số nguyên. Chứng minh. Giả sử g, f , h, S và V theo thứ tự là cạnh đáy, cạnh bên, chiều cao, diện tích toàn phần và thể tích của một hình chóp tứ giác đều và chúng là những số nguyên. Ta có f = r V = g 3 h; S = g2 + 2g r h2 + 2 ; h2 + 4 : g 2 2 g 2 Vì g, S là số nguyên nên 2g q h 2 + g 2 là số nguyên. Kí hiệu 4 x = 2g2 r h2 + 4 ; y = g3: g 2
  • 46. Viết thuê đề tài giá rẻ trọn gói - KB Zalo/Tele : 0973.287.149 Luanvanmaster.com – Cần Kham Thảo - Kết bạn Zalo/Tele : 0973.287.149 43 Dễ thấy x; y 2 N . Ta có x 2 y2 = 4g4 22 + g 2 g 6 = 4h 2 g 4 = (2g 2 h) 2 4 là số chính phương, vì V = g 3 2h 2 N . Do đó g 2 2 N . Mặt khác, g2 x2 + y2 = 4g4h2 + 2g6 = 4g4 h2 + = 4g4 f 2 = (2g2 f )2: Vì 2g 2 f 2 N nên x 2 + y 2 là số chính phương. Song điều này mâu thuẫn với Định lí 2.3.4. Ta có điều cần chứng minh. Bài toán 2.3.6. Chứng minh rằng phương trình x 4 y 4 = z 2 không có nghiệm nguyên dương. Chứng minh. Ta sẽ chứng minh bài toán này bằng phương pháp phản chứng. Giả sử (x; y; z) là nghiệm nguyên dương của phương trình đã cho và d = (x; y). Ta có x = dx0; y = dy0; (x0; y0) = 1: Thay vào phương trình cho ta d 4 (x0 4 y0 4 ) = z 2 suy ra d 2 j z: Đặt z = d 2 z0 suy ra x0 4 y0 4 = z0 2 , tức là (x0 2 y0 2 )(x0 2 + y0 2 ) = z0 2 : Nếu x0 và y0 chẵn lẻ khác nhau thì (x0 2 y0 2 ; x0 2 + y0 2 ) = 1 bởi vì nếu d1 j (x0 2 y0 2 ) và d1 j (x0 2 + y0 2 ) thì d1 j 2y0 2 và d1 j 2x0 2 . Vậy d1 = 1 (do d1 lẻ). Do đó x0 2 y 2 0 = a 2 ; x0 2 y 2 0 = b 2 , trái với Định lí 2.3.4. Nếu x và y cùng lẻ thì x2 +y2 = 2a; x2 +y2 = 2b Suy ra x2 = a+b, y2 = ab. 0 0 0 0 0 0 0 0 Ta có 4ab = z 2 suy ra ab = z 2 . Vì (x ; y ) = 1 nên (a; b) = 1. Do đó a = a 2 , b = b 2 . 0 1 0 0 1 1
  • 47. Viết thuê đề tài giá rẻ trọn gói - KB Zalo/Tele : 0973.287.149 Luanvanmaster.com – Cần Kham Thảo - Kết bạn Zalo/Tele : 0973.287.149 Vậy a 2 + b 2 = x 2 , a 2 b2 = y2 , và điều này mâu thuẫn với Định lí 2.3.4. 110110
  • 48. Viết thuê đề tài giá rẻ trọn gói - KB Zalo/Tele : 0973.287.149 Luanvanmaster.com – Cần Kham Thảo - Kết bạn Zalo/Tele : 0973.287.149 44 Bài toán 2.3.7. Chứng minh rằng phương trình x 4 + y 4 = z 2 không có nghiệm nguyên dương. Chứng minh. Giả sử trái lại, phương trình đã cho có nghiệm nguyên dương. Gọi (x0; y0) là nghiệm nguyên dương sao cho tổng x là nhỏ nhất. Lập luận tương tự như trong chứng minh Định lí 2.3.4 ta thấy (x0; y0) = 1. Như vậy (x0 2 ; y 2 0; z0) là một bộ ba số Pythagoras nguyên thuỷ. Không làm giảm tổng quát ta giả sử x0 lẻ. Theo Định lí 2.3.3 tồn tại m, n với (m; n) = 1 sao cho n x = m 2 n 2 ; y = 2mn;z0 = m 2 + n 2 : (2.24) Vì (m; n) = 1 nên từ (2.24) ta có (x0; n; m) là một bộ ba số Pythagoras nguyên thuỷ. Vậy lại theo Định lí 2.3.3 tồn tại a, b với (a; b) = 1 sao cho n x0 = a 2 b 2 ; n = 2ab;m = a 2 + b 2 : (2.25) Ta có y 2 0s = 2mn = 4abm suy ra abm = y 2 1 (ở đây y0 = 2y1) suy ra a = a 2 1, b = b 2 1, m = m 2 1. Thay vào (2.25) ta có m 2 1 = a 4 1 + b 4 1. Mặt khác a 4 1 + b 4 1 = m 2 1 = m < m 2 + n 2 = z0z 2 0 = x0 4 + y 4 0: Điều này trái với cách chọn (x0; y0). Vậy bài toán được chứng minh. Nhận xét 2.3.8. Qua chứng minh 2.3.4. và lời giải của các Bài toán 2.3.5, 2.3.6, 2.3.7, ta có thể nhận thấy một phương pháp chung sau đây trong lí thuyết phương trình Diophantine: Để chứng minh một phương trình Diophantine đã cho không có nghiệm nguyên dương, ta hãy giả sử nó có và khi đó sẽ có một nghiệm nhỏ nhất (theo một nghĩa nào đó). Sau đó ta sẽ cố gắng kiến thiết một nghiệm nhỏ hơn nghiệm nhỏ nhất và như vậy ta dẫn đến mâu thuẫn.
  • 49. Viết thuê đề tài giá rẻ trọn gói - KB Zalo/Tele : 0973.287.149 Luanvanmaster.com – Cần Kham Thảo - Kết bạn Zalo/Tele : 0973.287.149 45 Chương 3 Liên phân số và ứng dụng trong phương trình Diophantine